125
1. 法规 法规 法规 法规 1..(1-1) Determine the CCAR Part 121 landing minimums for the VOR/DME RWY 02 approach at Chengdu Inti Airport.PIC time 94 hours. A.(续考题正文,Airplane Vso maximum certificated weight 105 knots,VREF approach speed 140 knots )__MDH150m,VIS2400m. B.MDH120m,VIS1600m. C.MDH100m,VIS1600m. A X 2..(1-1) The PIC of PIL 10 has 87.5 hours and 26 landings as PIC in the B-767, while operating under Part 121. The PIC has 1,876 hours and 298 landings, as PIC in the L-1011 under Part 121. A.(续考题正文, What are the minimums for the VOR/DME RWY 02 approach at Chengdu Inti Airport for the PIC?)__MDH150m,VIS2400m. B.MDH120m,VIS1600m. C.MDH100m,VIS1600m. B X 3...(Refer to Figure 1-2) The PIC of PTL 55 has 75 hours and 30 landings as PIC in the B-747, while operating under Part 121. The PIC has 759 hours and 312 landings, as PIC, in the B-777 while operating under Part 121. A.(续考题正文 What are the minimums for the ILS RWY 02 approach at ZUCK, for the PIC?)_DH95mVIS1600 B.DH65mRVR600 C.DH65mRVR550 B X 4..(Refer to Figures 1-3) The PIC of TNA 90 has 49 hours and 102 landings as PIC in the MD90 while operating under Part 121. The PIC also has 959 hours and 246 landings, as PIC, in the B-737 while operating under Part 121. A.(续考题正文 What are the minimums for the ILS/DME RWY 18 approach at ZSSS, for this PIC?)_DA63m,RVR550m. B.DA93m,VIS1600m. C.DA110m,VIS1600m B X 5..(Refer to Figures 1-4) The PIC on TNA 90 (CAT C aircraft operated under CCAR Part 121) has not flown 100 hours as PIC in the MD90. What are the minimums while flying the ILS RWY 36R to land at ZBAA? A.DH20',VIS800m. B.DH200',RVR550m. C.DH300',VIS1600m. C X 6..(Refer to Figures 1-4) The weather at ZBTJ goes below the PICs minimums and TNA 90 (a CAT C aircraft operating under FAR Part 121 ) diverts to the alternate ZBAA. Upon arrival at ZBAA, TNA 90 is cleared for A.(续考题正文 an ILS/DME RWY 36R approach. The PIC has less than 100 hours of PIC time in the MD 90. What are the landing

1500 questions Very Important.pdf

  • Upload
    chanute

  • View
    47

  • Download
    3

Embed Size (px)

Citation preview

Page 1: 1500 questions Very Important.pdf

1. 法规法规法规法规

1..(1-1) Determine the CCAR Part 121 landing minimums for the VOR/DME

RWY 02 approach at Chengdu Inti Airport.PIC time 94 hours.

A.(续考题正文,Airplane Vso maximum certificated weight 105 knots,VREF

approach speed 140 knots )__MDH150m,VIS2400m.

B.MDH120m,VIS1600m.

C.MDH100m,VIS1600m.

A X

2..(1-1) The PIC of PIL 10 has 87.5 hours and 26 landings as PIC in the B-767,

while operating under Part 121. The PIC has 1,876 hours and 298 landings, as

PIC in the L-1011 under Part 121.

A.(续考题正文,What are the minimums for the VOR/DME RWY 02 approach at

Chengdu Inti Airport for the PIC?)__MDH150m,VIS2400m.

B.MDH120m,VIS1600m.

C.MDH100m,VIS1600m.

B X

3...(Refer to Figure 1-2) The PIC of PTL 55 has 75 hours and 30 landings as

PIC in the B-747, while operating under Part 121. The PIC has 759 hours and

312 landings, as PIC, in the B-777 while operating under Part 121.

A.(续考题正文 What are the minimums for the ILS RWY 02 approach at ZUCK,

for the PIC?)_DH95m,VIS1600

B.DH65m,RVR600

C.DH65m,RVR550

B X

4..(Refer to Figures 1-3) The PIC of TNA 90 has 49 hours and 102 landings as

PIC in the MD90 while operating under Part 121. The PIC also has 959 hours

and 246 landings, as PIC, in the B-737 while operating under Part 121.

A.(续考题正文 What are the minimums for the ILS/DME RWY 18 approach at

ZSSS, for this PIC?)_DA63m,RVR550m.

B.DA93m,VIS1600m.

C.DA110m,VIS1600m

B X

5..(Refer to Figures 1-4) The PIC on TNA 90 (CAT C aircraft operated under

CCAR Part 121) has not flown 100 hours as PIC in the MD90. What are the

minimums while flying the ILS RWY 36R to land at ZBAA?

A.DH20',VIS800m.

B.DH200',RVR550m.

C.DH300',VIS1600m.

C X

6..(Refer to Figures 1-4) The weather at ZBTJ goes below the PICs minimums

and TNA 90 (a CAT C aircraft operating under FAR Part 121 ) diverts to the

alternate ZBAA. Upon arrival at ZBAA, TNA 90 is cleared for

A.(续考题正文 an ILS/DME RWY 36R approach. The PIC has less than 100

hours of PIC time in the MD 90. What are the landing

Page 2: 1500 questions Very Important.pdf

minimums?)_DH20',VIS800m.

B.DH200',RVR550m.

C.DH300',VIS1600m.

C X

7..A Category II ILS pilot authorization, when originally issued, is normally

limited to

A.Category II operations not less than 1600 RVR and a 150-foot DH.

B.pilots who have completed an FAA-approved Category II training program.

C.Category II operations not less than 1200 RVR and a 100-foot DH.

A X

8..A commercial pilot has a type rating in a B-727 and B-737. A flight test is

completed in a B-747 for the Airline Transport Pilot Certificate. What pilot

privileges may be exercised regarding these airplanes?

A.Commercial - B-737; ATP - B-727 and B-747.

B.ATP - B-747; Commercial - B-727 and B-737.

C.ATP - B-747, B-727, and B-737.

C X

9..A commercial pilot has DC-3 and DC-9 type ratings. A flight test is completed

for an Airline Transport Pilot Certificate in a B-727. What pilot privileges may be

exercised?

A.ATP - B-727 and DC-3; Commercial - DC-9.

B.ATP - B-727 only; Commercial - DC-9 and DC 3.

C.ATP - B-727, DC-3, and DC-9.

C X

10..A crewmember who has served as second-in-command on a particular type

airplane (e.g., B737-300), may serve as pilot-in-command upon completing

which training program?

A.Upgrade training.

B.Recurrent training.

C.Initial training.

A X

11..A dispatch release for a flag or domestic air carrier must contain or have

attached to it

A.minimum fuel supply and weather information for the complete flight.

B.trip number and weight and balance data.

C.weather information for the complete flight and a crew list.

A X

12..A domestic air carrier airplane lands at an intermediate airport at 1815Z.

The latest time it may depart without a specific authorization from an aircraft

dispatcher is

A.1945Z.

B.1915Z.

C.1845Z.

B X

Page 3: 1500 questions Very Important.pdf

13..A domestic air carrier flight has a delay while on the ground, at an

intermediate airport. How long before a redispatch release is required?

A.Not more than 1 hour.

B.Not more than 2 hours.

C.More than 6 hours.

A X

14..A domestic or flag air carrier shall keep copies of the flight plans, dispatch

releases, and load manifests for at least

A.3 months.

B.6 months.

C.30 days.

A X

15..A flag air carrier flight lands at an intermediate airport at 1805Z. The latest

time that it may depart without being redispatched is

A.2005Z.

B.1905Z.

C.0005Z.

C X

16..A passenger briefing by a crewmember shall be given, instructing

passengers on the necessity of using oxygen in the event of cabin

depressurization, prior to flight conducted above

A. FL200

B. FL240

C. FL250

C X

17..A person whose duties include the handing or carriage of dangerous articles

and/or magnetized materials must have satisfactorily completed an established

and approved training program within the preceding

A. 6 calendar months.

B. 12 calendar months.

C. 24 calendar months.

B X

18..A pilot flight crewmember, other than pilot in command, must have received

a proficiency check or line-oriented simulator training within the preceding

A.6 calendar months.

B.12 calendar months.

C.24 calendar months.

C X

19..A pilot in command must complete a proficiency check or simulator training

within the preceding

A.6 calendar months.

B.12 calendar months.

C.24 calendar months.

A X

Page 4: 1500 questions Very Important.pdf

20..A pilot, acting as second-in-command, successfully completes the

instrument competency check. How long does this pilot remain current if no

further IFR flights are made?

A.12 months.

B.90 days.

C.6 months.

C X

21..A turbojet-powered flag air carrier airplane is released to an airport which

has no available alternate. What is the required fuel reserve?

A. 2 hours at normal cruise speed in a no wind condition fuel consumption.

B. 2 hours at normal cruise fuel consumption.

C. 30 minutes, plus 10 percent of the total flight time.

B X

22..An air carrier may schedule a pilot to fly in an airplane, in any calendar

month, for no more than

A.80hours

B.90 hours

C.100hours

B X

23..An air carrier may schedule a pilot to fly in an airplane, in any calendar year,

for no more than

A.800hours

B.900 hours

C.1000hours

C X

24..An air carrier may schedule a pilot, on a four-pilot crew (including a

second-in-commander pilot), for flight deck duty during any 24

consecutive-hour period for not more than

A.17 hours

B.24 hours

C.25 hours

A X

25..An air carrier may schedule a pilot, on a three-pilot crew (including a

second-in-commander pilot), for flight deck duty during any 24

consecutive-hour period for not more than

A.10 hours

B.14 hours

C.16 hours

A X

26..An air carrier uses an airplane that is certified for operation with a flightcrew

of two pilots and one flight engineer. In case the flight engineer becomes

incapacitated

A.at least one other flight crewmember must be qualified to perform the flight

engineer duties.

Page 5: 1500 questions Very Important.pdf

B.one crewmember must be qualified to perform the duties of the flight

engineer.

C.one pilot must be qualified and have a flight engineer certificate to perform

the flight engineer duties.

A X

27..An aircraft dispatcher shall receive at least 24 consecutive hours of rest

during

A.every 7 consecutive days

B.any 7 consecutive days or the equivalent(等同) thereof within any calendar

month

C.each calendar week

B X

28..An airplane has seats for 149 passengers and eight crewmembers. What is

the number of flight attendants required with 97 passengers aboard?

A.Four

B.Three

C.Two

B X

29..An airplane has seats for 49 passengers and two crewmembers. What is

the number of flight attendants required with only 1 passenger aboard?

A.Two

B.One

C.Zero

B X

30..An applicant who is scheduled for a practical test for an airline transport

pilot certificate, in an approved flight simulator, is

A.required to have at least a current third-class medical certificate.

B.not required to have a medical certificate.

C.required to have a first-class medical certificate.

B X

31..An applicant who is taking a practical test for a type rating to be added to a

commercial pilot certificate, in an approved simulator, is

A.required to have a first-class medical certificate.

B.required to have a second-class medical certificate.

C.not required to have a medical certificate.

C X

32..By regulation, who shall provide the pilot in command of a domestic or flag

air carrier airplane information concerning weather, and irregularities of facilities

and services?

A.The aircraft dispatcher.

B.Air route traffic control center.

C.Director of operations.

A X

33..Category II ILS operations below 1600 RVR and a 150-foot DH may be

Page 6: 1500 questions Very Important.pdf

approved after the pilot in command has

A.10 takeoffs and landings in make and model and three Category II ILS

approaches in actual or simulated IFR conditions with 150-foot DH since the

beginning of preceding month, under 14 CFR parts 91 and 121.

B. at least six Category II approaches in IFR conditions with 100-foot DH within

the preceding 12 calendar months.

C.logged 100 hours' in make and model airplane under 14 CFR part 121 and

three Category II ILS approaches in actual or simulated IFR with 150-foot DH

since the beginning of the sixth preceding month.

C X

34..During a supplemental air carrier flight, who is responsible for obtaining

information on meteorological (气象) conditions?

A.Aircraft dispatcher.

B.Pilot in command.

C.Director of operations or flight follower.

B X

35..During emergency, can a pilot expect landing priority?

A.. yes. delay all aircraft in the air and give this emergency aircraft landing

priority.

B. yes. Give the priority to this emergency aircraft only when have conflict with

other aircraft or proximate the same expect landing time.

C. no. because the landing priority of aircraft arriving at a tower-controlled

airport is on "first-come, first-served" basis.

B X

36..Duty and rest period rules require that a flight crewmember

A.not be assigned to any duty with the air carrier during any required rest

period.

B.not be on duty aloft for more than 100 hours in any 30-day period.

C.be relieved of all duty for at least 24 hours during any 7 consecutive days.

A X

37..Each crewmember shall have readily available for individual use on each

flight a

A. key to the flight deck door.

B. certificate holder's manual.

C. flashlight(手电筒) in good working order.

C X

38..Except during an emergency, when can a pilot expect landing priority?

A. When cleared for an IFR approach

B. When piloting a large, heavy aircraft.

C. In turn, on a firs-come, first serve basis.

C X

39..For a flag air carrier flight to be released to an island airport for which an

alternate airport is not available, a turbojet-powered airplane must have enough

fuel to fly to that airport and thereafter to fly

Page 7: 1500 questions Very Important.pdf

A. at least 2 hours at normal cruising fuel consumption.

B. for 3 hours at normal cruising fuel consumption(消耗).

C. back to the departure airport.

A X

40..For an airport with only one set of VOR approach, if the applicable

instrument visibility minimum is 2,000 meters, it may be listed as an alternate

airport only when the visibility is forecast to be at least

A.3000m.

B.3,600m.

C.2,800m.

B X

41..For an airport with only one set of VOR approach, if the authorized MDH is

100 meters, it may be listed as an alternate airport only when the ceiling is

forecast to be at least

A.220m.

B.150m.

C.160m.

A X

42..For an airport without air corridor(走廊), the approach controlled airspace

may extends laterally from the edge of Class D airspace to radius of

A.40 kilometers.

B.50 kilometers.

C.46 kilometers.

B X

43..For scheduled airline, pilots should fill in the box of TYPE OF FLIGHT with

letter

A.G.

B.S.

C.R.

B X

44..Group II aircraft are

A.Propeller driven aircraft

B.Turbojet aircraft

C.3/4 engine aircraft

B X

45..How dose deadhead transportation, going to or from a duty, affect the

computation of flight time limits for air carrier flight crewmember? It is

A.considered part of the rest period if the flight crew includes more than two

pilots.

B.considered part of the rest period for the flight engineers and navigators.

C.not considered to be part of a rest period.

C X

46..How long shall a supplemental air carrier or commercial operator retain a

record of the load manifest, airworthiness release, pilot route certification, flight

Page 8: 1500 questions Very Important.pdf

release, and flight plan?

A.1 month.

B.3 months.

C.12 months.

B X

47..How often must a crewmember actually operate the airplane emergency

equipment, after initial training?

A. 6 calendar months.

B. 12 calendar months.

C. 24 calendar months.

C X

48..If a B-747(400) airplane is dispatched from an airport that is below landing

minimums, what is the maximum distance that a departure alternate airport may

be located from the departure airport?

A.Not more than 2 hours at normal cruising speed with one engine inoperative.

B.Not more than 2 hours at normal cruising speed with one engine operating.

C.Not more than 2 hours at normal cruising speed in still air with one engine

inoperative.

C X

49..If a flag air carrier flight lands at an intermediate airport at 1845Z, and

experiences a delay, what is the latest time it may depart for the next airport

without a redispatch release?

A.1945Z.

B.2015Z.

C.0045Z.

C X

50..If a flight crewmember completes a required annual flight check in

December 1987 and the required annual recurrent flight check in January 1989,

the latter check is considered to have been taken in

A.November 1988.

B.December 1988.

C.January 1989.

B X

51..If a flight engineer becomes incapacitated during flight, who may perform

the flight engineer's duties?

A.The second in command only.

B.Any flight crewmember, if qualified.

C.Either pilot, if they have a flight engineer certificate.

B X

52..If an air carrier schedules a dispatcher for 12 hours of duty in a

24-consecutive-hour period, what action is required?

A.The dispatcher should be given a rest period of 24 hours at the end of the 13

hours

B.The dispatcher should be given a rest period of at least 8 hours at or before

Page 9: 1500 questions Very Important.pdf

the completion of 10 hours of duty

C.The dispatcher should refuse to be on duty 13hours as the CCAR part 121

limits the duty period to 10 consecutive hours

B X

53..If an intoxicated person creates a disturbance aboard and air carrier aircraft,

the certificate holder must submit a report, concerning the incident, to the

Administrator within

A. 10 days.

B. 24 hours.

C. 5 days.

C X

54..If it becomes necessary to shut down one engine on a domestic air carrier

three-engine turbojet airplane, the pilot in command

A. must land at the nearest suitable airport, in point of time, at which a safe

landing can be made.

B. may continue to the planned destination if approved by the company aircraft

dispatcher.

C. may continue to the planned destination if this is considered as safe as

landing at the nearest suitable airport.

C X

55..In a 24-hour consecutive period, what is the maximum time, excluding

briefing and debriefing, that an airline transport pilot may instruct other pilots in

air transportation service?

A.6 hours.

B.8 hours.

C.10 hours.

B X

56..It is the responsibility of the pilot and crew to report a near midair collision

(冲突) as a result of proximity(接近) of at least

A. 50 feet or less to another aircraft.

B. 500feet or less to another aircraft.

C. 1,000 feet or less to another aircraft.

B X

57..No one may operate in Class A airspace unless he has filed an IFR flight

plan and

A.reached the cruising IAS.

B.entered VFR weather conditions.

C.received an ATC clearance.

C X

58..person may not act as a crewmember of a civil aircraft if alcoholic

beverages have been consumed by that person within the preceding

A.8 hours.

B.12 hours.

C.24 hours.

Page 10: 1500 questions Very Important.pdf

A X

59..The "age 60 rule" of CCAR Part 121 applies to

A.any required pilot crewmember.

B.any flight crewmember.

C.the pilot in command only.

A X

60..The certificated air carrier and operators who must attach to, or include on,

the flight release form the name of each flight crewmember, flight attendant,

and designated pilot in command are

A.supplemental and commercial.

B.supplemental and domestic.

C.flag and commercial.

A X

61..The flight time limitations established for flight crewmembers include

A. only commercial flying in any flight crewmember position in which CCAR 121

operations are conducted.

B. all flight time, except military, in any flight crewmember position.

C. all commercial flying in any flight crewmember position.

C X

62..The flight time limitations established for flight crewmembers include

A.only pilots

B.pilots, navigators, flight engineers and batmen, but do not include flight

attendants.

C.all commercial flying in any flight crewmember position.

C X

63..The information required in the flight release for supplemental air carriers

and commercial operators that is not required in the dispatch release for flag

and domestic air carriers is the

A.weather reports and forecasts.

B.names of all crewmembers.

C.minimum fuel supply.

B X

64..The kinds of operation that a certificate holder is authorized to conduct are

specified in the

A.certificate holder's operations specifications.

B.application submitted for an Air Carrier or Operating Certificate, by the

applicant.

C.Air Carrier Certificate or Operating Certificate.

A X

65..The maximum duty time in 24 consecutive hours that a carrier may

schedule a pilot in a four pilot crew (including a second-in-commander pilot)

without a rest period is

A.19hours

B.20 hours

Page 11: 1500 questions Very Important.pdf

C.21hours

B X

66..The maximum duty time in 24 consecutive hours that a carrier may

schedule a pilot in a three pilot crew (including a second-in-commander pilot)

without a rest period is

A.16hours

B.17 hours

C.18 hours

A X

67..The maximum duty time in 24 consecutive hours that a carrier may

schedule a pilot in a two pilot crew without a rest period is

A.12hours

B.13 hours

C.14 hours

C X

68..The maximum flight time in 24 consecutive hours that a carrier may

schedule a pilot in a two pilot crew without a rest period is

A.8 hours

B.10 hours

C.12 hours

A X

69..The maximum number of consecutive hours of duty that an aircraft

dispatcher may be scheduled is

A.12hours

B.10hours

C.8hours

B X

70..The maximum number of hours a pilot may fly in 7 consecutive days without

any rest is

A.35 hours

B.32 hours

C.30 hours

A X

71..The required crewmember functions that are to be performed in the event of

an emergency shall be assigned by the

A. pilot in command.

B. air carrier's chief pilot.

C. certificate holder.

C X

72..The training required by flight crewmembers who have not qualified and

served in the same capacity on another airplane of the same group (e.g.,

turbojet powered) is

A.upgrade training.

B.transition training.

Page 12: 1500 questions Very Important.pdf

C.initial training.

C X

73..The training required for crewmembers or dispatchers who have been

qualified and served in the same capacity on other airplanes of the same group

is

A.difference training.

B.transition training.

C.upgrade training.

B X

74..To be eligible(合格) for the practical test for the renewal(恢复) of a

Category II authorization, what recent(最近的) instrument approach

experience is required?

A.Within the previous 6 months, six ILS approaches, three of which may be

flown to the Category I DH by use of an approach coupler.

B.Within the previous 6 months, six ILS approaches flown by use of an

approach coupler to the Category I DH.

C.Within the previous 12 calendar months, three ILS approaches flown by use

of an approach coupler to the Category II DH.

A X

75..To exercise ATP privileges a pilot must hold

A.a third-class medical certificate.

B.a second-class medical certificate.

C.a first-class medical certificate.

C X

76..To satisfy the minimum required instrument experience for IFR operations,

a pilot must accomplish during the past 6 months at least

A.six instrument approaches, holding, intercepting and tracking courses

through the use of navigation systems in an approved flight training

device/simulator or in the category of aircraft to be flown.

B.six instrument approaches, three of which must be in the same category and

class of aircraft to be flown, plus holding, intercepting and tracking courses in

any aircraft.

C.six instrument approaches and 6 hours of instrument time, three of which

may be in a glider.

A X

77..Under which condition is a flight engineer required as a flight crewmember

in CCAR Part 121 operations?

A.If the airplane is being flown on proving flights, with revenue cargo aboard.

B.If the airplane is powered by more than two turbine engines.

C.If required by the airplane's type certificate.

C X

78..What information must be contained in, or attached to, the dispatch release

for a domestic air carrier flight?

A.Departure airport, intermediate stops, destinations, alternate airports, and trip

Page 13: 1500 questions Very Important.pdf

number.

B.Names of all passengers on board and minimum fuel supply.

C.Cargo load, weight and balance data, and identification number of the

aircraft.

A X

79..What information must be included on a domestic air carrier dispatch

release?

A.Evidence that the airplane is loaded according to schedule, and a statement

of the type of operation.

B.Minimum fuel supply and trip number.

C.Company or organization name and identification number of the aircraft.

B X

80..What information must the pilot in command of a supplemental air carrier

flight or commercial operator carry to the destination airport?

A.Cargo and passenger distribution information.

B.Copy of the flight plan.

C.Names of all crewmembers and designated pilot in command.

B X

81..What instrument flight time may be logged by a second-in-command of an

aircraft requiring two pilots?

A.All of the time the second-in-command is controlling the airplane solely by

reference to flight instruments.

B.One-half the time the flight is on an IFR flight plan.

C.One-half the time the airplane is in actual IFR conditions.

A X

82..What is one of the requirements that must be met by an airline pilot to

re-establish recency of experience?

A.At least one landing must be made from a circling approach.

B.At least one full stop landing must be made.

C.At least one precision approach must be made to the lowest minimums

authorized for the certificate holder.

B X

83..What is the minimum number of flight attendants required on an airplane

having a passenger seating capacity of 238 with only 200 passengers aboard?

A.Five

B.Four

C.Two

A X

84..What is the minimum number of flight attendants required on an airplane

having a passenger seating capacity of 238 with only 40 passengers aboard?

A.Six

B.Five

C.Two

B X

Page 14: 1500 questions Very Important.pdf

85..What is the minimum number of flight attendants required on an airplane

with a passenger seating capacity of 333 when 296 passengers are aboard?

A.Seven

B.Six

C.Five

A X

86..What minimum condition is suggested for declaring(宣布) an emergency?

A. Anytime the pilot is doubtful(疑问) of a condition that could adversely affect

flight safety.

B. When fuel endurance or weather will require an en route or landing priority.

C. When distress conditions such as fire, mechanical failure, or structural

damage occurs.

A X

87..What recent experience is required to be eligible for the practical test for the

original issue of a Category II authorization?

A.Within the previous 6 months, six ILS approaches flown manually to the

Category I DH.

B.Within the previous 12 calendar months, six ILS approaches flown by use of

an approach coupler to the Category I or Category II DH.

C.Within the previous 6 months, six ILS approaches, three of which may be

flown to the Category I DH by use of an approach coupler.

C X

88..What requirement must be met regarding cargo that is carried anywhere in

the passenger compartment of an air carrier airplane?

A. The bin in which the cargo is carried may not be installed in a position that

restricts access to, or use of, any exit.

B. The bin in which the cargo is carried may not be installed in a position that

restricts access to, or use of, any aisle in the passenger compartment.

C. The container or bin in which the cargo is carried must be made of material

which is at least flash resistant.

B X

89..When a flag air carrier airplane lands at an intermediate airport at 1822Z,

what is the latest time it may continue a flight without receiving a redispatch

authorization?

A.1922Z.

B.1952Z.

C.0022Z.

C X

90..When a flight engineer is a required crewmember on a flight, it is necessary

for

A.one pilot to hold a flight engineer certificate and be qualified to perform the

flight engineer duties in an emergency.

B.the flight engineer to be properly certificated and qualified, but also at least

one other flight crewmember must be qualified and certified to perform flight

Page 15: 1500 questions Very Important.pdf

engineer duties.

C.at least one other flight crewmember to be qualified to perform flight engineer

duties, but a certificate is not required.

C X

91..When a pilot's flight time consists of 80 hours' pilot in command in a

particular type airplane, how does this affect the minimums for the destination

airport?

A.Has no effect on destination but alternate minimums are no less than

100m(MDH/DH) and VIS1600m.

B.Minimums are decreased by 30m(MDH/DH) and 800m(VIS).

C.Minimums are increased by 30m(MDH/DH) and 800m(VIS).

C X

92..When a speed adjustment is necessary to maintain separation, what

minimum speed may ATC request of a arrival aircraft operating below 3,000

meters?

A. Turbojet aircraft, a speed not less than 230 knots. Piton engine and

turboprop aircraft, a speed not less than 150 knots.

B. a speed not less than 210 knots; except 20 miles from threshold, not less

than 170 knots. Piston and turboprop aircraft. not less than 200 knots; except

20 flying miles from threshold, not less than 150 knots.

C. Not less than 250 knots

B X

93..When a temporary replacement is received for an airman's certificate, for

what maximum time is this document valid?

A.60 days.

B.90 days.

C.120 days.

C X

94..When a turbine-engine-powered airplane is to be ferried to another base for

repair of an inoperative engine, which operational requirement must be

observed?

A.Only the required flight crewmembers may be on board the airplane.

B.The existing and forecast weather for departure, en route, and approach must

be VFR.

C.No passengers except authorized maintenance personnel may be carried.

A X

95..When a type rating is to be added to an airline transport pilot certificate, and

the practical test is scheduled in an approved flight training device and/or

approved flight simulator, the applicant is

A.required to have at least a third-class medical certificate.

B.is not required to have a medical certificate.

C.required to have a first-class medical certificate.

B X

96..When an air carrier airplane with a seating capacity of 187 has 67

Page 16: 1500 questions Very Important.pdf

passengers on board, what is the minimum number of flight attendants

required?

A.Four

B.Three

C.Two

A X

97..When may a Category II ILS limitation be removed?

A.When three Cat II ILS approaches have been completed to a 150-foot

decision height and landing.

B.When six ILS approaches to Category II minimums and landing have been

completed in the past 6 months.

C.120 days after issue or renewal.

A X

98..When may a pilot descend below 100 feet above the touchdown zone

elevation during a Category II ILS instrument approach when only the approach

lights are visible?

A.After passing the visual descent point (VDP).

B.When the RVR is 1,600 feet or more.

C.When the red terminal bar of the approach light systems are in sight.

C X

99..Where are the routes listed that require special navigation equipment?

A.Certificate holder's Operations Specifications

B.International Aeronautical Information Manual

C.International Notices To Airmen

A X

100..Which dispatch requirement applies to an international air carrier that is

scheduled for a 10 hours IFR flight?

A.No alternate airport is required if the visibility at the destination airport is

desired to be at least 4,800m.

B.An alternate airport is required.

C.An alternate airport is not required if the ceiling at the destination airport is at

least 600m AGL.

B X

101..Which document includes descriptions of the required crewmember

functions to be performed in the event of an emergency?

A. Airplane Flight Manual.

B. Certificate holder's manual.

C. Pilot's Emergency Procedures Handbook.

B X

102..Which document specifically authorizes a person to operate an aircraft in a

particular geographic area?

A.Operations Specifications.

B.Operating Certificate.

C.Dispatch Release.

Page 17: 1500 questions Very Important.pdf

A X

103..Which documents are required to be carried aboard each domestic air

carrier flight?

A.Load manifest (or information from it) and flight release.

B.Dispatch release and weight and balance release.

C.Dispatch release, load manifest (or information from it), and flight plan.

C X

104..Which documents are required to be carried aboard each flag air carrier

flight?

A.Dispatch release, flight plan, and weight and balance release.

B.Load manifest, flight plan, and flight release.

C.Dispatch release, load manifest, and flight plan.

C X

105..Which flight plan requirement applies to an international air carrier that is

scheduled for a 10 hours IFR flight?

A.An alternate airport is required.

B.No alternate airport is required if the visibility at the destination airport is

desired to be at least 4,800m.

C.An alternate airport is not required if the ceiling at the destination airport is at

least 600m AGL.

A X

106..Which ground components are required to be operative for a Category II

approach in addition to LOC glide slope, marker beacons, and approach lights?

A.Radar and RVR.

B.RCLS and REIL

C.HIRL, TDZL, RCLS, and RVR.

C X

107..Which information must be contained in, or attached to, the dispatch

release for a flag air carrier flight?

A.Type of operation (e.g., IFR, VFR), trip number.

B. Total fuel supply and minimum fuel required on board the airplane.

C.Passenger manifest, company organization name, and cargo weight.

A X

108..Which is a definition of the term "crewmember"?

A.Only a pilot, flight engineer, or flight navigator assigned to duty in an aircraft

during flight time.

B.A person assigned to perform duty in an aircraft during flight time.

C.Any person assigned to duty in an aircraft during flight except a pilot or flight

engineer.

B X

109..Which is a definition of the term "flight crewmember"?

A.a pilot, flight engineer, or flight navigator assigned to duty in the aircraft during

flight time.

B.Any person assigned to perform duty in an aircraft during flight time.

Page 18: 1500 questions Very Important.pdf

C.Any person assigned to duty in an aircraft during flight except a pilot or flight

engineer.

A X

110..Which is one of the requirements that must be met by a required pilot flight

crewmember in re-establishing recency of experience?

A.At least one landing must be made with a simulated failure of the most critical

engine.

B.At least one ILS approach to the lowest ILS minimums authorized for the

certificate holder and a landing from that approach.

C.At least three landings must be made to a complete stop.

B X

111.Which operational requirement must be observed by a commercial operator

when ferrying a large, three-engine, turbojet-powered airplane from one facility

to another to repair an inoperative engine?

A.The Airplane Flight Manual must include procedures and performance data

which allow for the safe operation of such a flight.

B.The existing and forecast weather for departure, en route, and approach must

be VFR.

C.Some passengers may be carried.

A X

112..Which operational requirement must be observed by a commercial

operator when ferrying a large, three-engine, turbojet-powered airplane from

one facility to another to repair an inoperative engine?

A.The computed takeoff distance to reach V1 must not exceed 70 percent of

the effective runway length.

B.The initial climb cannot be over thickly-populated areas

C.The existing and forecast weather for departure, en route, and approach

must be VFR.

B X

113..Which passenger announcement(s) must be made after each takeoff?

A. Keep satety belts fastened while seated and no smoking in the aircraft

lavatories.

B. Passengers should keep seat belts fastened while seated.

C. How to use the passenger oxygen system and that there is a $1,000 fine for

tampering with a smoke detector.

B X

114..Who is responsible, by regulation, for briefing a domestic or flag air carrier

pilot in command on all available weather information?

A.Company meteorologist.

B.Aircraft dispatcher.

C.Director of operations.

B X

115..Who must the crew of a domestic or flag air carrier airplane be able to

communicate with, under normal conditions, along the entire route (in either

Page 19: 1500 questions Very Important.pdf

direction) of flight?

A.ARINC

B.Any FSS

C.Appropriate dispatch office

C X

116..With regard to flight crewmembers duties, which of the following

operations are considered to be in the "critical phases of flight"?

A. taxi, takeoff, landing, and all other operations conducted below 3,000 meters

MSL, including cruise flight.

B. descent, approach, landing, and taxi operations, irrespective of altitudes

MSL.

C. taxi, takeoff, landing, and all other operations conducted below 3,000 meters,

excluding cruise flight.

C X

2. 机载设备机载设备机载设备机载设备,,,,地面设备地面设备地面设备地面设备

TCAS I provides

A.Traffic and resolution advisories

B.Proximity warning

C.Recommended maneuvers to avoid conflicting traffic

B X

Which range of codes should a pilot avoid switching through when changing

transponder codes?

A. 0000 through 1000.

B. 7200 and 7500 series.

C. 7500, 7600, and 7700 series.

C X

Without Receiver Autonomous Integrity Monitoring (RAIM) capability, the

accuracy of the GPS derived

A. Altitude information should not be relied upon to determine aircraft altitude.

B. Position is not affected

C. Velocity information should be relied upon to determine aircraft groundspeed

A X

What functions are provided by ILS?

A. Azimuth, distance, and vertical angle.

B. Azimuth, range, and vertical angle.

C. Guidance, range, and visual information.

C X

To make all the route aircraft have the same zero altitude, the altimeter should

set on .

A.the standard air pressure of the mean sea level

B.the standard air pressure of the local station

C.the standard air pressure of the calibrated sea level

Page 20: 1500 questions Very Important.pdf

A X

What are the indications of Precision Approach Path Indicator (PAPI)?

A.High-white, on glidepath-red and white; low-red.

B.High-white, on glidepath-green; low-red.

C.High-white and green, on glidepath-green; low-red.

A X

What are the indications of the pulsating VASI?

A.High-pulsing white, on glidepath-green, low-pulsing red.

B.High-pulsing white, on glidepath-steady white, slightly below glide slope

steady red, low-pulsing red.

C.High-pulsing white, on course and on glidepath-steady white, off course but

on glidepath-pulsing white and red; low-pulsing red.

B X

What are the line check requirements for the pilot in command for a domestic

air carrier?

A.The line check is required every 12 calendar months in one of the types of

airplanes to be flown.

B.The line check is required only when the pilot is scheduled to fly into special

areas and airports.

C.The line check is required every 12 months in each type aircraft in which the

pilot may fly.

A X

What are the lowest Category IIIA minimum?

A.DH 50 feet and RVR 1,200 feet.

B.RVR 1,000 feet

C.RVR 700 feet.

C X

What aural and visual indications should be observed over an ILS inner

marker?

A. Continuous dots at the rate of six per second.

B. Continuous dashes at the rate of two per second.

C. Alternate dots and dashes at the rate of two per second.

A X

What can a pilot expect if the pitot system ram air input and drain hole are

blocked by ice?

A.The airspeed indicator may act as an altimeter.

B.The airspeed indicator will show a decrease with an increase in altitude

C.No airspeed indicator change will occur during climbs or descents

A X

What does the tri-color VASI consist of?

A.Three light bar; red, green, and amber.

B.One light projector with three colors; green, and amber.

C.Three glide slopes, each a different color; red, green, and amber.

B X

Page 21: 1500 questions Very Important.pdf

What facilities may be substituted for an inoperative middle marker during a

Category I ILS approach?

A.ASR and PAR.

B.The middle marker has no effect on straight-in minimums.

C.Compass locator, PAR, and ASR.

B X

What does the blue radial line on the airspeed indicator of a light, twin-engine

airplane represent?

A.Maximum single-engine rate of climb.

B.Maximum single-engine angle of climb.

C.Minimum controllable airspeed for single-engine operation.

A X

What does the precision Approach Path Indicator (PAPI) consist of?

A.Row of four lights parallel to the runway; red, white, and green.

B.Row of four lights perpendicular to the runway; red and white.

C.One light projector with two colors; red and white.

B X

What does the pulsating VASI consist of?

A.Three-light system, two pulsing and one steady.

B.Two-light projectors, one pulsing and one steady.

C.One –light project, pulsing white when above glide slope or red when more

than slightly below glide slope, steady white when on glide slope, steady red for

slightly below glide path.

C X

What is corrected altitude (approximate true altitude)?

A.Pressure altitude corrected for instrument error.

B.Indicated altitude corrected for temperature variation from standard.

C.Density altitude corrected for temperature variation from standard.

B X

What is the advantage of a three-bar VASI?

A.Pilots have a choice of glide angles

B.A normal glide angle is afforded both high and low cockpit aircraft.

C.The three-bar VASI is mush more visible and can be used at a greater height.

B X

What is the advantage of HIRL or MIRL on an IFR runway as compared to a

VFR runway?

A.Lights are closer together and easily distinguished from surrounding lights

B.Amber lights replace white on the last 2,000 feet of runway for a caution zone

C.Alternate red and white lights replace the white on the last 3,000 feet of

runway for caution zone

B X

What is the purpose of Runway End Identifier Lights (REIL)?

A.Identification of a runway surrounded by a preponderance of other lighting.

B.Identification of the touchdown zone to prevent landing short.

Page 22: 1500 questions Very Important.pdf

C.Establish visual descent guidance information during an approach.

A X

What type navigation system is inertial Navigation System (INS)? A navigation

computer which provides position

A.From information by compass, airspeed, and an input of wind and variation

data.

B.From radar-type sensors that measure ground speed and drift angles.

C.By signals fro, self-contained gyros and accelerometers.

C X

What type service should normally be expected from an En Route Flight

Advisory Service?

A. Weather advisories pertinent to the type of flight, intended route of flight, and

altitude

B. Severe weather information ,changes in flight plans, and receipt of position

reports.

C. Radar vectors for traffic separation, route weather advisories, and altimeter

settings.

A X

When is DME required for an instrument flight?

A.At or above 24000 feet MSL if VOR navigational equipment is required

B.In terminal radar service areas

C.Above 12,500 feet MSL

A X

When is the course deviation indicator (CDI) considered to have a full-scale

deflection?

A.When the CDI deflects from full-scale left to full-scale tight, or vice versa.

B.When the CDI deflects from the center of the scale to full-scale left or right.

C.When the CDI deflects from half-scale left to half-scale right, or vice versa.

B X

When making an approach to a narrower-than-usual runway, without VASI

assistant, the pilot should be aware that the approach

A.attitude may be higher than it appears.

B.attitude may be lower than it appears.

C.may result in leveling off too high and landing hard.

B X

When making an approach to a wider-than-usual runway, without VASI

assistant, there is a tendency

A.to fly a lower-than -normal approach.

B.to fly a higher-than -normal approach.

C.to fly a normal approach.

B X

When must an air carrier airplane be DME equipped?

A. In class e airspace for all IFR or VFR on top operations.

B.Whenever VOR navigational receivers are required

Page 23: 1500 questions Very Important.pdf

C.For flights at or above FL 180

B X

When using cabin heat in light single-engined aircraft it should be remembered

that

A.the use of full cabin heat during take-off can degrade the aircrafts take-off

performance

B.there is a possibility that carbon monoxide can be introduced directly into the

cockpit through exhaust leaks in the heart exchanger

C.cabin heat should be used in cruising flight only, and never in climbs or

descents

B X

Which color on a tri-color VASI is a "high" indication?

A.Red

B.Amber

C.Green.

B X

Which color on a tri-color VASI is an "on course" indication?

A.Red

B.Amber

C.Green

C X

Which component associated with the ILS is identified by the first two letters of

the localizer identification group?

A.Inner marker.

B.Middle compass locator.

C.Outer compass locator.

C X

Where provided, an automatic terminal information service (ATIS) will assist

operations prior to_______________________.

A. the commencement of final approach

B. enter TMA

C. the commencement of initial approach

A X

Which "rule-of-thumb" may be used to approximate the rate of descent required

for a 3 glide path?

A.5times groundspeed in knots.

B. 8times groundspeed in knots.

C.10 times groundspeed in knots.

A X

Which airplanes are required to be equipped with a ground proximity warning

glide slope deviation alerting system?

A.All turbine powered airplanes

B.Passenger -carrying turbine-powered airplanes only

C.Large turbine-powered airplanes only

Page 24: 1500 questions Very Important.pdf

A X

Which equipment requirement must be met by an air carrier that elects to use a

dual inertial Navigation System (INS) on a proposed flight?

A.The dual system must consist of two operative INS units.

B.A dual VORTAC/ILS system may be substituted for an inoperative INS.

C.Only one INDS is required to be operative, if a Doppler Radar is substituted

for the other INS.

C X

Which facility may be substituted for the middle marker during a Category I ILS

approach?

A.VOR/DME FIX

B.Surveillance radar

C.Compass locator

C X

which rule applies to the use of the cockpit voice recorder erasure feature?

A.All recorded information may be erased, except for the last 30 minutes prior

to landing.

B.Any information more than 30 minutes old may be erased

C.All recorded information may be erased, unless the NTSB needs to be

notified of an occurrence.

B X

Which pressure is defined as station pressure?

A.Altimeter setting.

B.Actual pressure at field elevation.

C.Station barometric pressure reduced to sea level.

B X

Within what frequency range dies the localizer transmitter of the ILS operate?

A.108.10 to 118.10 MHZ

B.108.10 to 111.95 MHZ

C.108.10 to 117.95 MHZ.

B X

(Refer to Figure 2-10) This is an example of

A.An ILS Critical Area Holding Position Sign

B.A runway Boundary Sign

C.An ILS Critical Area Boundary Sign

C X

(Refer to Figure 2-11) What is the runway distance remaining at "A" for a

daytime takeoff in runway 9?

A.1,000 feet

B.1,500 feet

C.2,000 feet

A X

(Refer to Figure 2-12) what is the runway distance remaining at "A" for a

nighttime takeoff on runway 9?

Page 25: 1500 questions Very Important.pdf

A.1,000 feet

B.2,000 feet

C.2,500 feet

B X

(Refer to Figure 2-12) What is the runway distance remaining at "B" for a

daytime takeoff on runway 9?

A.2,000 feet

B.2,500 feet

C.3,000 feet

C X

(Refer to Figure 2-12) What is the runway distance remaining at "B" for a

daytime takeoff on runway 9?

A.2,000 feet

B.2,500 feet

C.3,000 feet

C X

(Refer to Figure 2-12) What is the runway distance remaining at "D" for a

daytime takeoff on runway 9?

A.500 feet

B.1,000feet

C.1,500 feet

B X

(Refer to Figure 2-13) What is the runway distance remaining at "A" for a

nighttime takeoff on runway 9?

A.2,000 feet

B.3,000 feet

C.3,500 feet.

B X

(Refer to Figure 2-13) What is the runway distance remaining at "B" for a

nighttime takeoff on runway 9?

A.1,000 feet

B.2,000 feet

C.2,500 feet.

B X

(Refer to Figure 2-13) What is the runway distance remaining at "C" for a

nighttime takeoff on runway 9?

A.1,000 feet

B.1,500 feet

C.1,800 feet.

A X

(Refer to Figure 2-13) What is the runway distance remaining at "E" for a

daytime takeoff on runway 9?

A.1,500 feet

B.2,000 feet

Page 26: 1500 questions Very Important.pdf

C.2,500 feet

B X

(Refer to Figure 2-1and 2-2) To which aircraft position dose HSI presentation

"A" respond?

A.1

B.8

C.11

A X

(Refer to Figure 2-1and 2-2) To which aircraft position dose HSI presentation

"C" respond?

A.6

B.7

C.12

C X

(Refer to Figure 2-1and 2-2) To which aircraft position dose HSI presentation

"D" correspond?

A.4

B.15

C.17

C X

(Refer to Figure 2-1and 2-2) To which aircraft position dose HSI presentation "E

"or respond?

A.5

B.5

C.15

B X

(Refer to Figure 2-3) On which radial is the aircraft as indicated by the NO.1

NAV?

A.R-175

B.R-165

C.R-345

C X

(Refer to Figure 2-3) What is the lateral displacement in degrees from the

desired radial on the NO.2 NAV?

A.1?

B.-2?

C.-4?

C X

(Refer to Figure 2-3) What is the lateral displacement of the aircraft in nautical

miles from the radial selected on the No.1 NAV?

A.5.0NM

B.7.5NM

C.10.0NM

A X

Page 27: 1500 questions Very Important.pdf

(Refer to Figure 2-3) Which OBS selection on the No.2 NAV would center the

CDI and change the ambiguity indication to a TO ?

A.166

B.346

C.354

C X

(Refer to Figure 2-6) What is the magnetic bearing TO the station as indicated

by illustration 4?

A.285

B.055

C.235

B X

(Refer to Figure 2-6) Which RMI illustration indicates the aircraft is southwest of

the station and moving closer TO the station?

A.1

B.2

C.3

A X

(Refer to Figure 2-6) Which RMI illustration indicates the aircraft to be flying

outbound on the magnetic bearing of 235 FROM the station ?(wind050 at 20

knots)

A.2

B.3

C.4

B X

(Refer to Figure 2-9) This sign, which faces the runways and is visible to the

pilot, indicates

A.A point at which the pilot should contact ground control without being

instructed by tower

B.A point at which the aircraft will be clear of runway

C.The point at which the emergency arresting gear is stretched across the

runway.

B X

(Refer to Figures 2-4 and 2-5) To which aircraft position does HSI presentation

"B" correspond?

A.11

B.5 and 13

C.7and 11

B X

(Refer to Figures 2-4 and 2-5) To which aircraft position does HSI presentation

"H" correspond?

A.8

B.1

C.2

Page 28: 1500 questions Very Important.pdf

B X

(Refer to Figures 2-4 and 2-5) To which aircraft position does HSI presentation

"I" correspond?

A.4

B.12

C.11

C X

(Refer to Figures 2-4 and 2-5)To which aircraft position does HSI presentation

"A" correspond?

A.9and6

B.9 only

C.6 only

A X

(Refer to Figures 2-7a and 2-8) Which displacement from the localizer and glide

slope at the 1.9 NM point is indicated?

A.710 feet to the left of the localizer centerline and 140 feet below the glide

slope.

B.710 fee to the right of the localizer centerline and 140 feet above the glide

slope.

C.430 feet to the right of the localizer centerline and 28 feet above the glide

slope.

B X

(Refer to Figures 2-7b and 2-8) Which displacement from the localizer

centerline and glide slope at the 1,3000-foot point from the runway is indicated?

A.21 feet below the glide slope and approximately 320 feet to the right of the

runway centerline.

B.28 feet above the glide slope and approximately 250 feet to the left of the

runway centerline.

C.21 feet above the glide slope and approximately 320 feet to the left of the

runway centerline.

C X

A function of the Minimum Equipment List is to indicate required items which

A.Are required to operative for overwater passenger air carrier flights.

B.May be inoperative for a one-time ferry flight of a large airplane to a

maintenance base.

C.May be inoperative prior to beginning a flight in an aircraft.

C X

A Land and Hold Short Operations (LAHSO) clearance, that the pilot accepts:

A.Must be adhered to.

B.Does not preclude rejected landing.

C.Precludes a rejected landing.

B X

A pilot approaching to land a turbine-powered aircraft on a runway served by a

VASI shall

Page 29: 1500 questions Very Important.pdf

A.Not use the VASI unless a clearance for a VASI approach is received.

B.Use the VASI only when weather conditions are below basic VFR.

C.Maintain an altitude at or above the glide slope until a lower altitude is

necessary for a safe landing.

C X

A pilot of a high-performance airplane should be aware that flying a

steeper-than-normal VASI glide slope angle may result in

A. A hard landing.

B. Increased landing rollout.

C. Landing short of the runway threshold.

B X

Airport information signs, used to provide destination or information, have

A.Yellow inscriptions on a black background

B.White inscriptions on a black background

C.Black inscriptions on a yellow background

C X

An air carrier airplane's airborne radar must be in satisfactory operating

condition prior to dispatch, if the flight will be

A.Conducted under VFR conditions at night with scattered thunderstorms

reported en route

B.Carrying passengers, but not if it is "all cargo"

C.Conducted IFR, and ATC is able to radar vector the flight around areas of

weather

A X

An air carrier operates a flight in VFR over-the-top conditions. What radio

navigation equipment is required to be a dual installation?

A.VOR

B.VOR and ILS

C.VOR and DME

A X

An air carrier that elects to use an inertial Navigational System (INS) must meet

which equipment requirement prior to takeoff on a proposed flight?

A.The INS system must consist of two operative INS units.

B.Only one INS is required to be operative, if a Doppler Radar is substituted for

other INS.

C.A dual VORTAC/ILS system may be substituted for an inoperative INS.

B X

Below FL180, en route weather advisories should be obtained from an FSS on

A.122.1MHz

B.122.0MHZ

C.123.6MHz

B X

During an en route descent in a fixed-thrust and fixed-pitch attitude

configuration, both the ram air input and drain hole of the pilot system become

Page 30: 1500 questions Very Important.pdf

completely blocked by ice. What airspeed indication can be expected?

A.Increase in indicated airspeed.

B.Decrease in indicated airspeed.

C.Indicated airspeed remains at the value prior to icing

B X

En route at FL270, the altimeter is set correctly. On descent, a pilot fails to set

the local altimeter setting of 30.57.If the field elevation is 650feet,and the

altimeter is functioning properly, what will it indicate upon landing?

A.585feet

B.1.300feet

C.Sea level

C X

For the purpose of testing the flight recorder system.

A.A minimum of 1 hour of the oldest recorded data must be erased to get a valid

test

B.A total of 1 hour of the oldest recorded data accumulated at the time of testing

may be erased

C.A total of no more than 1 hour of recorded data may be erased

B X

GPS instrument approach operations, outside the United States, must be

authorized by

A. The FAA-approved aircraft flight manual (AFM) or flight manual supplement

B. A sovereign country or government unit

C. The FAA Administrator only

B X

Holding line markings at the intersection of taxiways and runways consist of

tour lines (two solid and two dashed) that extend across the width of taxiway,

These lines are

A.White in color and the dashed lines are nearest the runway

B.Yellow in color and the dashed lines are nearest the runway

C.Yellow in color and the solid lines are nearest the runway

B X

Holding position signs have

A.White inscriptions on a red background

B.Red inscriptions on a white background

C.Yellow inscriptions on a red background

A X

How can a pilot identify a lighted heliport at night?

A.Green yellow and white beacon light

B.White and red beacon light with dual flash of the white

C.Green and white beacon light with dual flash of the white

A X

How long is cockpit voice recorder and flight recorder data kept, in the event of

accident or occurrence resulting in terminating the flight?

Page 31: 1500 questions Very Important.pdf

A.60days

B.90days

C.30days

A X

Identify REIL.

A.Amber lights for the first 2,000 feet of runway.

B.Green lights at the threshold and red lights at far end of runway.

C.Synchronized flashing lights laterally at each side of the runway threshold.

C X

Identify runway remaining lighting on centerline lighting systems

A.Amber lights from 3000 feet to 1,000 feet, then alternate red and white lights

to the end

B.Alternate red and white lights from 3,000 feet to 1,000 feet, then red lights to

the end

C.Alternate red and white lights from3,000 feet to the end of the runway

B X

Identify touchdown zone lighting (TDZL)

A.Two rows of transverse light bars disposed symmetrically about the runway

centerline

B.Flush centerline lights spaced at 50-foot intervals extending through the

touchdown zone

C.Alternate white and green centerline lights extending from 75 feet from the

threshold through the touchdown zone

A X

Identigy the runway distance remaining markers

A.Signs with increments of 1,000 feet distance remaining

B.Red markers laterally placed across the runway at 3.000 feet from the end

C.Yellow marker laterally placed across the runway with signs on the side

denoting distance to end

A X

If a required instrument on a multiengine airplane becomes inoperative, which

document dictates whether the flight may continue en route?

A. An approved Minimum Equipment list for the airplane.

B. Original dispatch release.

C. Certificate holder's manual.

C X

If an air carrier airplane is flying IFR using a single ADF navigation receiver and

the ADF equipment fails, the flight must be able to

A.Proceed safely to a suitable airport using VOR aids and complete an

instrument approach by use of the remaining airplane system.

B.Continue to the destination airport by means of dead reckoning navigation

C.Proceed to a suitable airport using VOR aids complete an instrument

approach and land.

A X

Page 32: 1500 questions Very Important.pdf

If an air carrier airplane's airborne radar is inoperative and thunderstorms are

forecast along the proposed route of flight, an airplane may be dispatched only

A.When able to climb and descend VFR and maintain VFR/OT en route

B.In VFR conditions

C.In day VFR conditions

C X

If an airborne checkpoint is used to check the VOR system for IFR operations,

the maximum bearing error permissible is

A.Plus or minus 6

B.Plus 6 or minus 4

C.Plus or minus4?

A X

If both the ram air input and drain hole of the pitot system are blocked by ice,

what airspeed indication can be expected?

A.No variation of indicated airspeed in level flight if large power changes are

made

B.Decrease of indicated airspeed during a climb

C.Constant indicated airspeed during a descent

A X

If Receiver Autonomous Integrity Monitoring (RAIM) is not available when

setting up for GPS approach , the pilot should

A. Continue to the MAP and hold until the satellites are recaptured .

B. Proceed as to cleared to the IAF and hold until satellite reception is

satisfactory ,

C. Select another type of navigation aid.

C X

In conducting Land and Hold Short Operations (LAHSO), the pilot should have

readily available:

A.The published Available Landing Distance (ALD), landing performance of the

aircraft, and slope of all LAHSO combinations at the destination airport.

B.The published runway length and slope for all LAHSO combinations at the

airport of intended landing.

C.The landing performance of the aircraft, published Available Landing

Distance (ALD) for all LAHSO combinations at the airport of intended landing,

plus the forecast winds.

A X

3.空气动力学空气动力学空气动力学空气动力学

One purpose of high-lift devices is to

A.- increase the load factor.

B.delay stall.

C.increase airspeed.

B X

What is a feature of air movement in a high pressure area?

Page 33: 1500 questions Very Important.pdf

A. Ascending from the surface high to lower pressure at higher altitudes.

B. Descending to the surface and then outward.

C. Moving outward from the center at high altitudes and into the center at the

surface.

B X

The Krueger flap extends from the leading edge of the wing, increasing its

A.angle of attack

B.camber

C.energy

B X

The leading edge of an advancing cold air mass is .

A. warm front.

B. stationary front.

C. cold front.

C X

The primary purpose of high-lift devices is to increase the

A.-L/Dmax.

B.-Lift at low speeds.

C.-Drag and reduce airspeed.

B X

To avoid the wingtip vortices of a departing jet airplane during takeoff, the pilot

should

A. Lift off at a point well past the jet airplane's flight path.

B. Climb above and stay upwind of the jet airplane's flight path.

C. Remain below the flight path of the jet airplane.

B X

Upon which factor does wing loading during a level coordinated turn in smooth

air depend?

A.Rate of turn

B.Angle of bank

C.True airspeed

B X

What action is expected of an aircraft upon landing at a controlled airport?

A. continue taxiing in the landing direction until advised by the tower to switch to

ground control frequency.

B. exit the runway at the nearest suitable taxiway and remain on tower

frequency until instructed otherwise

C. exit the runway at the nearest suitable taxiway and switch to ground control

upon crossing the taxiway holding lines.

B X

What action is required prior to takeoff if snow is adhering to the wings of an air

carrier airplane?

A. sweep off as much snow as possible and the residue must be polished

smooth.

Page 34: 1500 questions Very Important.pdf

B. Assure that the snow is removed from the airplane.

C. Add 15 knots to the normal VR speed as the snow will blow off.

B X

What action shall the pilot in command take if it becomes necessary to shut

down one of the two engines on an air carrier airplane?

A. Land at the airport which the pilot considers to be as safe as the nearest

suitable airport in point of time.

B. Land at the nearest suitable airport in point of time at which a safe landing

can be made.

C. Land at the nearest airport, including military what has a crash an rescue

unit.

B X

What action should a pilot take when a clearance appears to be contrary to a

regulation?

A.Request a clarification from the ATC.

B.Read the clearance back entirely.

C.Do not accept the clearance.

A X

What action should a pilot take when he is uncertain of the meaning of an ATC

clearance?

A.Do not accept the clearance.

B.Request a clarification from the ATC.

C.Read the clearance back entirely.

B X

What action should be taken by the pilot in command of a transport category

airplane if the airborne weather radar becomes inoperative en route on an IFR

flight for which weather reports indicate possible thunderstorms?

A.Request radar vector from ATC to the nearest suitable airport and land

B.Proceed in accordance with the approved instructions and procedures

specified in the operationsmanual for such an event

C.Return to the departure airport if the thunderstorms have not been

encountered. and there is enough fuel remaining

B X

What action should be taken if one of the two VHF radios fail while IFR in

controlled airspace?

A. Notify ATC immediately.

B. Squawk 7600.

C. Monitor the VOR receiver

A X

What affects indicated stall speed?

A. Weight, load factor, and power

B. Load factor , angle of attack ,and power

C. Angle of attack, weight, and airspeed

A X

Page 35: 1500 questions Very Important.pdf

What effect does the leading edge slot in the wing have on performance?

A. Decrease profile drag.

B. Changes the stalling angle of attack to a higher angle.

C. Decelerates the upper surface boundary layer air.

B X

What effect would a light crosswind have on the wingtip vortices generated by a

large airplane that has just taken off?

A. The upwind vortex will tend to remain on the runway longer than the

downwind vortex.

B. A crosswind will rapidly dissipate the strength of both vortices.

C. The downwind vortex will tend to remain on the runway longer than the

upwind vortex.

A X

What flight condition should be expected when an aircraft leaves ground effect?

A.An increase in induced drag requiring a higher angle of attack.

B.A decrease in parasite drag permitting a lower angle of attack.

C.An increase in dynamic stability.

A X

What is a characteristic of longitudinal instability?

A.Pitch oscillations becoming progressively greater.

B.Bank oscillations becoming progressively greater.

C.Aircraft constantly tries to pitch down.

A X

What is a purpose of flight spoilers?

A.-Increase the camber of the wing.

B.-Reduce lift without increasing airspeed.

C.-Direct airflow over the top of the wing at high angles of attack.

B X

What is load factor?

A.Lift multiplied by the total weight.

B.Lift subtracted from the total weight.

C.Lift divided by the total weight.

C X

What is movement of the center pressure when the wingtips of a swept wing

airplane are stalled first?

A.inward and aft

B.inward and forward

C.outward and forward

B X

What is one disadvantage of a swept wing design?

A.The wing root stalls prior to the wingtip section.

B.The wingtip section stalls prior to the wing root.

C.Severe pitch down moment when the center of pressure shifts forward.

B X

Page 36: 1500 questions Very Important.pdf

What is the condition known as when gusts cause a swept wing-type airplane to

roll in one direction while yawing in the other?

A.Porpoise.

B.Wingover.

C.Dutch roll

C X

What is the effect on total drag of an aircraft if the airspeed decreases in level

flight below that speed for maximum L/D?

A.Drag increases because of increased induced drag.

B.Drag increases because of increased parasite drag.

C.Drag decreases because of lower induced drag.

A X

What is the primary function of the leading edge flaps in landing configuration

during the flare before touchdown?

A.prevent flow separation.

B.decrease rate of sink.

C.increase profile drag.

A X

What is the principal advantage of a sweepback design wing over a straight

wing design?

A.The critical Mach number will increase significantly.

B.Sweepback will increase changes in the magnitude of force coefficients due

to compressibility.

C.Sweepback will accelerate the onset of compressibility effect.

A X

What is the purpose of a control tab?

A.Move the flight controls in the event of manual reversion.

B.Reduce control forces by deflecting in the proper direction to move a primary

flight control.

C.Prevent a control surface from moving to a full-deflection position due to

aerodynamic forces.

A X

What is the purpose of a servo tab?

A.Move the flight controls in the event of manual reversion.

B. Reduce control forces by deflecting in the proper direction to move a primary

flight control.

C. Prevent a control surface from moving to a full-deflection position due to

aerodynamic forces.

B X

What is the purpose of an elevator trim tab?

A.Provide horizontal balance as airspeed is increased to allow hands-off flight.

B.Adjust the speed tail load for different airspeeds in flight allowing neutral

control forces.

C.Modify the downward tail load for various airspeeds in flight eliminating

Page 37: 1500 questions Very Important.pdf

flight-control pressures.

C X

What is the reason for variations in geometric pitch along a propeller or rotor

blade?

A.It permits a relatively constant angle of attack along its length when in cruising

flight.

B.It prevents the portion of the blade near the hub or root from stalling during

cruising flight.

C.It permits a relatively constant angle of incidence along its length when in

cruising flight.

A X

what is the relationship between induced and parasite drag when the gross

weight is increased?

A.parasite drag increases more than induced drag

B. induced drag increases more than parasite drag

C. both parasite and induced drag are equally increased.

B X

What is the relationship of the rate of turn with the radius of turn with a constant

angle of bank but increasing airspeed?

A.-rate will decrease and radius will increase.

B.Rate will decrease and radius will decrease.

C.Rate and radius will increase.

A X

What is the result of a shock-induced separation of airflow occurring near the

wing root of a swept wing aircraft?

A.-A high-speed stall and sudden pitch up

B.A severe moment or "tuck under"

C.-Severe porpoising

B X

What taxi action is expected of an aircraft upon landing at a controlled airport?

A. continue taxiing in the landing direction.

B. exit the runway at the any suitable taxiway, or make a 180 degrees turn on

the runway if overshot a expect taxiway.

C. exit the runway at the nearest suitable taxiway without delay.

C X

What true airspeed and angle of attack should be used to generate the same

amount of lift as altitude is increased?

A. The same true airspeed and angle of attack

B. A higher true airspeed for any given angle of attack

C. A lower true airspeed and higher angle of attack

B X

What will be the ratio between airspeed and lift if the angle of attack and other

factors remain constant and airspeed is doubled? lift will be

A.The same.

Page 38: 1500 questions Very Important.pdf

B.Two times greater.

C.Four times greater.

C X

When are inboard ailerons normally used?

A.Low-speed flight only.

B.High-speed flight only.

C.Low-speed and high-speed flight.

C X

Which condition would INITIALLY cause the indicated airspeed and pitch to

increase and sink rate to decrease?

A. Sudden increase in a headwind component.

B. Sudden decrease in a headwind component.

C. Tailwind which suddenly increase in velocity.

A X

Which direction from the primary control surface does a servo tab move?

A.Some direction.

B.Opposite direction.

C.Remains fixed for all positions.

B X

Which direction from the primary control surface dose an anti-servo tab move?

A.Same direction.

B.Opposite direction.

C.Remains fixed for all positions

A X

Which direction from the primary control surface dose an elevator adjustable

trim tab move when the control surface is moved?

A.Same direction.

B.Opposite direction.

C.Remains fixed for all positions.

C X

Which flight conditions of a large jet airplane create the most severe flight

hazard by generating wingtip vortices of the greatest strength?

A. Heavy, slow, gear and flaps up.

B. Heavy, slow, gear and flaps down.

C. Heavy, fast, gear and flaps down.

A X

Which is a purpose of ground spoilers?

A. Reduce the wings lift upon landing

B. Aid in rolling an airplane into turn.

C. Increase the rate of descent with out gaining airspeed.

A X

Which is a purpose of leading-edge slats on high-performance wings?

A.-Decrease lift at relative slow speeds.

B.-Improve aileron control during low angles of attack.

Page 39: 1500 questions Very Important.pdf

C.-Direct air from the high pressure area under the leading edge along the top

of the wing.

C X

Which is a purpose of the leading-edge flaps?

A.Increase the camber of the wing.

B.Reduce lift without increasing airspeed.

C.Direct airflow over the top of the wing at high angles of attack.

A X

Which is a purpose of wing-mounted vortex generators?

A. Reduce the drag caused by supersonic flow over portions of the wing

B. Increase the onset of drag divergence and aid in aileron effectiveness at high

speed

C. Break the airflow over the wing so the stall will progress from the root out to

the tip of the wing

A X

Which is true regarding the forces acting on an aircraft in a steady-state

descent? The sum of all

A.-upward forces is less than the sum of all downward forces.

B.-rearward forces is greater than the sum of all forces.

C.-forward forces is equal to the sum of all rearward forces.

C X

Which of the following are considered primary flight controls?

A.Tabs.

B.Flaps.

C.Outboard ailerons.

C X

Which of the following is not the way of increasing MCRIT in jet transport

designs

A. give the wing a lower camber

B. increase wing sweep.

C.add slats

C X

Which statement is true concerning the wake turbulence produced by a large

transport aircraft?

A. Vortices can be avoided by flying 300 feet below and behind the flight path of

the generating aircraft.

B. The vortex characteristics of any given aircraft may be altered by extending

the flaps or changing the speed.

C. Wake turbulence behind a propeller-driven aircraft is negligible because jet

engine thrust is a necessary factor in the formation of vortices.

B X

while maintaining a constant angle of bank and altitude in a coordinated turn, an

increase in airspeed will

A.decrease the rate of turn resulting in a decreased load factor.

Page 40: 1500 questions Very Important.pdf

B.decrease the rate of turn resulting in no change in load factor

C.increase the rate of turn resulting in no change in load factor

B X

While making prolonged constant rate turns under IFR conditions, an abrupt

head movement

A.(续正文, can create the illusion of rotation on an entirely different axis. This is

known as )autokinesis.

B.Coriolis illusion.

C.the leans.

B X

Wingtip vortices created by large aircraft tend to

A. Sink below the aircraft generating the turbulence.

B. Rise from the surface to traffic pattern altitude.

C. Accumulate and remain for a period of time at the point where the takeoff roll

began.

A X

Why do some airplanes equipped with inboard/outboard ailerons use the

outboards for slow flight only?

A.-Increased surface area provides greater controllability with flap extension.

B.-Aerodynamic loads on the outboard ailerons tend to twist the wingtips at high

speeds.

C.-Locking out the outboard ailerons in high-speed flight provides variable flight

control feel.

B X

Why is it necessary to increase back elevator pressure to maintain altitude

during a turn? To compensate for the

A.loss of the vertical component of lift

B.loss of the horizontal component of lift and the increase in centrifugal force

C.rudder deflection and slight opposite aileron throughout the turn

A X

Why must the angle of attack be increased during a turn to maintain altitude?

A.Compensate for loss of vertical component of lift

B.Increase the horizontal component of lift equal to the vertical component

C.Compensate for increase in drag

A X

Within what mach range dose transonic fight regimes usually occur?

A.-.50 to .75 mach

B.-.75 to .1.20 mach

C.-1.20 to 2.50 mach

B X

Airflow separation over the wing can be delayed by using vortex generators.

A.Directing high pressure air over the top of the wing or flap through slots and

making the wing surface smooth.

B.Directing a suction over the top of the wing or flap through slots and making

Page 41: 1500 questions Very Important.pdf

the wing surface smooth.

C. Making the wing surface rough and/or directing high pressure air over the top

of the wing or flap through slots.

C X

At what Mach range does de subsonic flight range normally occur?

A.Below.75 Mach.

B.From.75 to 1.20 Mach.

C.From.1.20 to 2.5. Mach.

A X

At which speed will increasing the pitch attitude cause an airplane to climb?

A.Low speed.

B.High speed.

C.Any speed.

B X

By changing the angle of attack of a wing, the pilot can control the airplane's

A.lift, gross weight, and drag.

B.lift, airspeed, and drag.

C.lift and airspeed, but not drag.

B X

Describe dynamic longitudinal stability

A.Motion about the longitudinal.

B.Motion about the lateral axis.

C.Motion about the vertical axis.

B X

For a given angle of bank, the load factor imposed on both the aircraft and pilot

in a coordinated constant-altitude turn

A.is directly related to the airplane's gross weight.

B.varies with the rate of turn.

C.is constant

C X

For which purpose may flight spoilers be used?

A.Reduce the wings' lift upon landing.

B.Increase the rate of descent without increasing aerodynamic drag.

C.Aid in longitudinal balance when rolling an airplane into a turn.

A X

How can an airplane produce the same lift in ground effect as when out of

ground effect?

A.The same angle of attack.

B.A lower angle of attack.

C.A higher angle of attack.

B X

How can the pilot increase the rate of turn and decrease the radius at the same

time?

A.- Steepen the bank and increase airspeed.

Page 42: 1500 questions Very Important.pdf

B. Steepen the bank and decrease airspeed.

C.Shallow the bank and increase airspeed.

B X

How does the wake turbulence vortex circulate around each wingtip?

A. Inward, upward, and around the wingtip.

B. Counterclockwise when viewed from behind the aircraft.

C. Outward, upward, and around the wingtip.

C X

Identify the type stability if the aircraft attitude remains in new position after the

controls have been neutralized.

A.Negative longitudinal static stability

B. Neutral longitudinal dynamic stability

C.-Neutral longitudinal static stability

C X

Identify the type stability if the aircraft attitude tends to move farther from its

original position after the controls have been neutralized

A.-Negative static stability

B.-Positive static stability

C.-Negative dynamic stability

A X

Identify the type stability if the aircraft attitude tends to return to its original after

the controls have been neutralized.

A.Positive dynamic stability.

B.Positive static stability.

C.Neutral dynamic stability.

B X

If an aircraft level turns at an angle of bank of 30 degree , the load factor is

A.1Gs.

B.2Gs.

C.1.4Gs

C X

If an aircraft with a gross weight of 2000 pounds were subjected to a total load

of 6000 pounds in flight the load factor would be

A.2Gs.

B.3Gs.

C.9Gs

B X

If no corrective action is taken by the pilot as angle of bank is increased, how is

the vertical component of lift and sink rate affected?

A.Lift increases and the sink rate increases

B.Lift decreases and the sink rate decreases

C.Lift decreases and the sink rate increases

C X

If severe turbulence is encountered, which procedure is recommended?

Page 43: 1500 questions Very Important.pdf

A.Maintain a constant altitude.

B.Maintain a constant attitude.

C.Maintain constant airspeed and altitude.

B X

If the airplane attitude remains in a new position after the elevator control is

pressed forward and released, the airplane displays

A.-neutral longitudinal static stability.

B.-positive longitudinal static stability.

C.-neutral longitudinal dynamic stability.

A X

If you take off behind a heavy jet that has just landed, you should plan to lift off

A. Prior to the point where the jet touched down.

B. Beyond the point where the jet touched down.

C. At the point where the jet touched down and on the upwind edge of the

runway.

B X

4.性能性能性能性能

1..Maximum range performance of a turbojet aircraft is obtained by which

procedure as aircraft weight reduces?

A.Increasing speed or altitude.

B.Increasing altitude or decreasing speed.

C.Increasing speed or decreasing altitude.

B X

2..Which of the following phraseology is suitable for a pilot reporting a braking

action on well condition to ATC?

A. braking action is "fair"

B. braking action is "good"

C. braking action is "100%"

B X

3..Which of the following phraseology is suitable for a pilot reporting a braking

action to ATC, when there is almost no the braking action?

A. braking action is "fair"

B. braking action is "zero"

C. braking action is "nil"

C X

4..Which of the following phraseology is suitable for a pilot reporting a braking

action to ATC?

A. braking action is "fair"

B. braking action is "zero"

C. braking action is "100%"

A X

5..The critical altitude of a supercharged reciprocating engine is

A.The highest altitude where the mixture can be leaned to best power ratio.

Page 44: 1500 questions Very Important.pdf

B.The highest altitude at which a desired manifold pressure can be obtained.

C.The altitude at which maximum allowable BMEP can be obtained.

B X

6..The maximum speed during takeoff that the pilot may abort the takeoff and

stop the airplane within the accelerate-stop distance is

A.V2.

B.VEF.

C.V1.

C X

7..The minimum speed during takeoff, following a failure of the critical engine at

VEF, at which the pilot may continue the takeoff and achieve the required height

above the takeoff surface within the takeoff distance, is indicated by symbol

A.V2min .

B.V1.

C.VLOF.

B X

8..The most important restriction to the operation of turbojet or turboprop

engines is

A.Limiting compressor speed.

B.Limiting exhaust gas temperature.

C.Limiting torque.

B X

9..The symbol for the speed at which the critical engine is assumed to fail

during takeoff is

A.V2.

B.V1.

C.VEF.

C X

10..The symptoms of hyperventilation(换气过度) are

A.quite different to those of hypoxia

B.quite similar to those of hypoxia

C.profuse sweating

B X

11.。Under normal operating conditions, which combination of MAP and RPM

most likely lead to detonation in high performance reciprocating engines?

A.High RPM and low MAP.

B.Low RPM and high MAP.

C.High RPM and high MAP.

B X

12..Under what condition is VMC the highest?

A.Gross weight is at the maximum allowable value.

B.CG is at the most rearward allowable position.

C.CG is at the most forward allowable position.

B X

Page 45: 1500 questions Very Important.pdf

13..Under which condition during the landing roll are the main wheel brakes at

maximum effectiveness?

A.When wing lift has been reduced.

B.At high ground speeds.

C.When the wheels are locked and skidding.

A X

14..What characterizes a transient compressor stall?

A.Loud, steady roar accompanied by heavy shuddering.

B.Sudden loss of thrust accompanied by a loud whine.

C.Intermittent "bang," as backfires and flow reversals take place.

C X

15..What effect will an increase in altitude have upon the available equivalent

shaft horsepower (ESHP) of a turboprop engine?

A.Lower air density and engine mass flow will cause a decrease in power.

B.Higher propeller efficiency will cause an increase in usable power (ESHP)

and thrust.

C.Power will remain the same but propeller efficiency will decrease.

A X

16..What effect would a change in air density have on gas-turbine-engine

performance?

A.As air density decreases, thrust increases.

B.As air density increases, thrust increases.

C.As air density increases, thrust decreases.

B X

17..What effect would a change in altitude have on the thrust output of

gas-turbine-engine?

A.As altitude increases, thrust decreases.

B.As altitude increases, thrust increases.

C.As altitude decreases, thrust decreases.

A X

18..What effect would a change in altitude have on the thrust output of

gas-turbine-engine?

A.As altitude increases, thrust decreases.

B.As altitude increases, thrust increases.

C.As altitude decreases, thrust decreases.

A X

19..What effect would a change in ambient temperature have on

gas-turbine-engine performance?

A.As temperature increases, thrust decreases.

B.As temperature increases, thrust increases.

C.As temperature decreases, thrust decreases.

A X

20..What effect would a change in ambient temperature or air density have on

gas-turbine-engine performance?

Page 46: 1500 questions Very Important.pdf

A.As air density decreases, thrust increases.

B.As temperature increases, thrust increases.

C.As temperature increases, thrust decreases.

C X

21..What effect does an increase in airspeed have on a coordinated turn while

maintaining a constant angle of bank an altitude?

A.The rate of turn will decrease resulting in a decreased load factor.

B.The rate of turn will increase resulting in an increased load factor.

C.-The rate of turn will decrease resulting in no changes in load factor.

C X

22..What effect does an uphill runway slope have upon takeoff performance?

A.Increases takeoff distance.

B.Decreases takeoff speed.

C.Decreases takeoff distance.

A X

23..What effect does high relative humidity have upon the maximum power

output of modern aircraft engines?

A.Neither turbojet nor reciprocating engines are affected.

B.Reciprocating engines will experience a significant loss of BHP.

C.Turbojet engines will experience a significant loss of thrust.

B X

24..What effect does landing at high elevation airports have on ground speed

with comparable conditions relative to temperature, wind, and airplane weight?

A.Higher than at low elevation.

B.Lower than at low elevation.

C.The same as at low elevation.

A X

25..What effect does landing at high temperature have on ground speed with

comparable conditions relative to airport elevation, wind, and airport elevation?

A.Higher than at low temperature.

B.Lower than at low temperature.

C.The same as at low temperature.

A X

26..What effect does landing with increased weight have on ground speed with

comparable conditions relative to temperature, wind, and airport elevation?

A.Higher than at low weight.

B.Lower than at low weight.

C.The same as at low weight.

A X

27..What does the term of Equivalent shaft horsepower (ESHP) of a turboprop

engine describe?

A.shaft horsepower and jet thrust.

B.jet thrust only.

C.propeller thrust only.

Page 47: 1500 questions Very Important.pdf

A X

28..What does the throttle opening of a turbo-charged-reciprocating engine

control?

A.Supercharger gear ratio.

B.Exhaust gas discharge.

C.The desired manifold pressure.

C X

29..What effect, if any, does high ambient temperature have upon the thrust

output of a turbine engine?

A.Thrust will be reduced due to the decrease in air density.

B.Thrust will remain the same, but turbine temperature will be higher.

C.Thrust will be higher because more heat energy is extracted from the hotter

air.

A X

30..What effective runway length is required for a turbojet-powered airplane at

the destination airport if the runways are forecast to be wet or slippery at the

ETA?

A.70 percent of the actual runway available, from a height of 50 feet over the

threshold.

B.115 percent of the runway length required for a dry runway.

C.115 percent of the runway length required for a wet runway.

B X

31..What indicates that a compressor stall has developed and become steady?

A.Strong vibrations and loud roar.

B.Occasional loud "bang" and flow reversal.

C.Complete loss of power with severe reduction in airspeed.

A X

32..What is an area identified by the term "clearway"?

A.centrally located about the extended centerline and under airport authorities,

which does not contain obstructions and can be considered when calculating

takeoff performance.

B.An area, at least the same width as the runway, capable of supporting an

airplane during a normal takeoff.

C.An area beyond the takeoff runway, which is able to support the airplane, for

use in decelerating the airplane during an aborted takeoff.

A X

33..What is an area identified by the term "stopway"?

A.An area, at least the same width as the runway, capable of supporting an

airplane during a normal takeoff.

B.An area designated for use in decelerating an aborted takeoff.

C.An area, not as wide as the runway, capable of supporting an airplane during

a normal takeoff.

B X

34..What is the best method of speed reduction if hydroplaning is experienced

Page 48: 1500 questions Very Important.pdf

on landing?

A.Apply full main wheel braking only.

B.Apply nosewheel and main wheel braking alternately and abruptly.

C.Apply aerodynamic braking to the fullest advantage.

C X

35..What is the correct symbol for minimum unstick speed?

A.VMU.

B.VMD.

C.VFC.

A X

36..What is the correct symbol for takeoff decision speed?

A.V2.

B.V1.

C.VLOF.

B X

37..What is the correct symbol for the Minimum Control Speed, Ground?

A.VMC.

B.VMCG.

C.VMCA.

B X

38..What is the definition of the critical altitude of a supercharged-reciprocating

engine?

A.Highest altitude where the mixture can be leaned to best power ratio.

B.The highest altitude at which a desired manifold pressure can be obtained.

C.The altitude at which maximum allowable BMEP can be obtained.

B X

39..what is the free stream mach number which produce first of local sonic

flow?

A.supersonic mach number

B.transonic mach number

C.critical mach number

C X

40..What is the highest speed possible without supersonic flow over the wing?

A.Initial buffet speed.

B.Critical Mach number.

C.Transonic index.

B X

41..What is the name of a plane beyond the end of a runway which does not

contain obstructions and can be considered when calculating takeoff

performance of turbine-powered aircraft?

A.Clearway.

B.Stop way.

C.Obstruction clearance plane.

A X

Page 49: 1500 questions Very Important.pdf

42..What is the resulting performance loss when one engine on a twin-engine

airplane fails?

A.Reduction of cruise airspeed by 50 percent.

B.Reduction of climb by 50 percent or more.

C.Reduction of all performance by 50 percent.

B X

43..What performance is characteristic of flight at maximum L/D in a

propeller-driven airplane?

A.Maximum range and distance glide.

B.Best angle of climb.

C.Maximum endurance.

A X

44..What performance should a pilot of a light, twin-engine airplane be able to

maintain at Vmc ?

A.Heading.

B.Heading and altitude.

C.Heading, altitude, and ability to climb 50 ft/min.

A X

45..What should a pilot do to maintain "best range" airplane performance when

a headwind is encountered?

A.Increase speed.

B.Maintain speed.

C.Decrease speed.

A X

46..Where is the critical altitude of a supercharged-reciprocating engine?

A.The highest altitude at which a desired manifold pressure can be obtained.

B.Highest altitude where the mixture can be leaned to best power ratio.

C.The altitude at which maximum allowable BMEP can be obtained.

A X

47..Which parameter is used to measure the power output of a reciprocating

engine?

A.RPM.

B.Exhaust gas temperature.

C.Manifold pressure.

C X

48..Which performance factor decreases as airplane gross weight increases,

for a given runway?

A.Critical engine failure speed.

B.Rotation speed.

C.Accelerate-stop distance.

A X

49..Which place in the turbojet engine is subjected to the highest temperature?

A.Compressor discharge.

B.Fuel spray nozzles.

Page 50: 1500 questions Very Important.pdf

C.Turbine inlet.

C X

50..Which condition reduces the required runway for takeoff?

A.Higher-than-recommended airspeed before rotation.

B.Lower-than-standard air density.

C.Increased headwind component.

C X

51..Which condition has the effect of reducing critical engine failure speed?

A.Slush on the runway or inoperative antiskid.

B.Low gross weight.

C.High density altitude.

A X

52..Where will the highest temperature in a turbojet engine occur?

A.Turbine inlet.

B.Compressor discharge.

C.Fuel spray nozzles.

A X

53..Which airplane performance characteristics should be recognized during

takeoff when encountering a tailwind shear that increases in intensity?

A. loss of, or diminished, airspeed performance

B. decreased takeoff distance

C. increased climb performance immediately after takeoff

A X

54..Which is a definition of V2 speed?

A.Takeoff decision speed.

B.Takeoff safety speed.

C.Minimum takeoff speed.

B X

55..Which is the correct symbol for the minimum steady-flight speed or stalling

speed in a specific configuration?

A.VS.

B.VS1.

C.VSO.

B X

56..Which is the correct symbol for the stalling speed or the minimum steady

flight speed at which the airplane is controllable?

A.VSO.

B.VS.

C.VS1.

B X

57..Which maximum range factor increases as weight decreases?

A.Angle of attack.

B.Altitude.

C.Airspeed.

Page 51: 1500 questions Very Important.pdf

B X

58..Which speed symbol indicates the maximum speed for operating the

landing gear of an airplane?

A.VLE.

B.VMO/MMO.

C.VLO/MLO.

C X

59..Which type of compressor stall has the greatest potential(潜能) for severe

engine damage?

A.Intermittent "backfire" stall.

B.Transient "backfire" stall.

C.Steady, continuous flow reversal stall.

C X

60.。Which term describes the hydroplaning, which occurs when an airplane's

tire is effectively held off a smooth runway surface by steam(蒸汽) generated

by friction?

A.Reverted rubber hydroplaning.

B.Dynamic hydroplaning.

C.Viscous hydroplaning.

A X

61..With the increase of temperature, the airplane's rolling distance will

____when takeoff and landing.

A. remain unchanged

B. increase

C. reduce

B X

62..[Refer to Figure 4-25.] How many feet will remain after landing on a

6,000-foot dry runway with reversers inoperative at 120,000 pounds gross

weight?

A.3,500 feet.

B.2,750 feet.

C.2,150 feet.

A X

63..[Refer to Figure 4-25.] How many feet will remain after landing on a

7,200-foot dry runway with spoilers inoperative at 118,000 pounds gross

weight?

A.4,200 feet.

B.4,500 feet.

C.4,750 feet.

B X

64..[Refer to Figure 4-25.] How much longer is the dry runway landing distance

using brakes only compared to using brakes and reversers at 110,000 pounds

gross weight?

A.1,000 feet.

Page 52: 1500 questions Very Important.pdf

B.500 feet.

C.300 feet.

C X

65..[Refer to Figure 4-25.] How much longer is the dry runway landing distance

using brakes only compared to using brakes and reversers at 114,000 pounds

gross weight?

A.1,150 feet.

B.500 feet.

C.300 feet.

C X

66..[Refer to Figure 4-25.] What is the maximum landing weight which will

permit stopping 1,500 feet short of the end of a 4,950-foot dry runway with

reversers and spoilers inoperative?

A.119,000 pounds.

B.136,000 pounds.

C.139,000 pounds.

C X

67..[Refer to Figure 4-25.] What is the maximum landing weight which will

permit stopping 2,000 feet short of the end of a 5,400-foot dry runway with

reversers and spoilers inoperative?

A.117,500 pounds.

B.136,500 pounds.

C.140,500 pounds.

B X

68..[Refer to Figure 4-25.] What is the transition distance when landing on a dry

runway at a gross weight of 125,000 pounds?

A.1,200 feet.

B.850 feet.

C.400 feet.

C X

69..[Refer to Figure 4-25.] Which of the following configurations will result in the

shortest landing distance over a 50-foot obstacle to a dry runway?

A.Brakes and spoilers at 120,000 pounds gross weight.

B.Brakes and reversers at 115,000 pounds gross weight.

C.Brakes, spoilers, and reversers at 125,000 pounds gross weight.

C X

70..[Refer to Figure 4-25.] Which of the following configurations will result in the

shortest landing distance over a 50-foot obstacle to a dry runway?

A.Brakes and spoilers at 125,000 pounds gross weight.

B.Brakes and reversers at 115,000 pounds gross weight.

C.Brakes, spoilers, and reversers at 130,000 pounds gross weight.

C X

71..[Refer to Figure 4-26.] How many feet will remain after landing on a

6,000-foot wet runway with reversers inoperative at 110,000 pounds gross

Page 53: 1500 questions Very Important.pdf

weight?

A.2,100 feet.

B.2,650 feet.

C.3,000 feet.

C X

72..[Refer to Figure 4-26.] How many feet will remain after landing on a

7,000-foot wet runway with reversers inoperative at 110,000 pounds gross

weight?

A.4,000 feet.

B.4,300 feet.

C.4,500 feet.

A X

73..[Refer to Figure 4-26.] How much longer is the wet runway landing distance

using brakes only compared to using brakes, spoilers and reversers at 130,000

pounds gross weight?

A.1,000 feet.

B.500 feet.

C.300 feet.

A X

74..[Refer to Figure 4-26.] How much longer is the wet runway landing distance

using brakes only compared to using brakes, spoilers and reversers at 140,000

pounds gross weight?

A.650 feet.

B.1,300 feet.

C.1,050 feet.

C X

75..[Refer to Figure 4-26.] What is the maximum landing weight which will

permit stopping 1,000 feet short of the end of a 4,900-foot wet runway with

reversers and spoilers inoperative?

A.124,000 pounds.

B.129,500 pounds.

C.134,500 pounds.

A X

76..[Refer to Figure 4-26.] What is the transition distance when landing on a wet

runway at a gross weight of 135,000 pounds?

A.100 feet.

B.300 feet.

C.750 feet.

B X

77..[Refer to Figure 4-26.] Which of the following configurations will result in the

shortest landing distance over a 50-foot obstacle to a wet runway?

A.Brakes and spoilers at 120,500 pounds gross weight.

B.Brakes and reversers at 125,000 pounds gross weight.

C.Brakes, spoilers, and reversers at 135,000 pounds gross weight.

Page 54: 1500 questions Very Important.pdf

C X

78..[Refer to Figure 4-27.] What is the maximum landing weight which will

permit stopping 600 feet short of the end of a 4,600-foot icy runway?

A.128,000 pounds.

B.124,000 pounds.

C.120,000 pounds.

C X

79..[Refer to Figure 4-27.] What is the maximum landing weight which will

permit stopping 700 feet short of the end of a 5,200-foot icy runway?

A.124,000 pounds.

B.137,000 pounds.

C.108,000 pounds.

B X

80..[Refer to Figure 4-27.] What is the transition distance when landing on an

icy runway at a gross weight of 134,000 pounds?

A.400 feet.

B.950 feet.

C.1,350 feet.

A X

81..[Refer to Figure 4-27.] Which configuration will result in a landing distance

of 5,000 feet over a 50 foot obstacle to an icy runway?

A.Use of three reversers at 113,000 pounds gross weight.

B.Use of brakes and spoilers at 105,000 pounds gross weight.

C.Use of three reversers at 115,000 pounds gross weight.

A X

82..[Refer to Figure 4-27.] Which configuration will result in a landing distance

of 5,500 feet over a 50 foot obstacle to an icy runway?

A.Use of three reversers at 124,500 pounds gross weight.

B.Use of brakes and spoilers at 120,000 pounds gross weight.

C.Use of three reversers at 120,000 pounds gross weight.

A X

83..[Refer to Figure 4-27.] Which configuration will result in a landing distance

of 5,900 feet over a 50 foot obstacle to an icy runway?

A.Use of three reversers at 131,000 pounds gross weight.

B.Use of brakes and spoilers at 125,000 pounds gross weight.

C.Use of three reversers at 133,000 pounds gross weight.

C X

84..[Refer to Figure 4-28 and 4-29.] What approach speed and landing distance

will be needed when landing at a weight of 140,000 pounds with 15o of flaps?

A.123 knots and 3,050 feet.

B.138 knots and 3,050 feet.

C.153 knots and 2,050 feet.

B X

85..[Refer to Figure 4-28.] How much will landing distance be reduced by using

Page 55: 1500 questions Very Important.pdf

15o of flaps rather than 0o flaps at a landing weight of 105,000 pounds?

A.800 feet.

B.300 feet.

C.1,300 feet.

A X

86..[Refer to Figure 4-28.] How much will landing distance be reduced by using

15o of flaps rather than 0o flaps at a landing weight of 115,000 pounds?

A.500 feet.

B.800 feet.

C.2,700 feet.

B X

87..[Refer to Figure 4-28.] How much will landing distance be reduced by using

15o of flaps rather than 0o flaps at a landing weight of 143,000 pounds?

A.100 feet.

B.400 feet.

C.850 feet.

C X

88..[Refer to Figure 4-28.] What is the ground roll when landing with 15o of

flaps at a landing weight of 122,000 pounds? (ground roll equal landing

distance subtracting 1000ft)

A.1,750 feet.

B.2,200 feet.

C.2,750 feet.

A X

89..[Refer to Figure 4-28.] What is the ground roll when landing with 5o of flaps

at a landing weight of 107,500 pounds?

A.1,750 feet.

B.2,000 feet.

C.2,350 feet.

B X

90..[Refer to Figure 4-28.] What is the ground roll when landing with 5o of flaps

at a landing weight of 142,500 pounds?

A.1,750 feet.

B.2,100 feet.

C.2,500 feet.

C X

91..[Refer to Figure 4-29.] What is the change of total drag for a 140,000-pound

airplane when configuration is changed from flaps 25o, gear down, to flaps 0o,

gear up, at a constant airspeed of 165 knots?

A.9,500 pounds.

B.10,400 pounds.

C.11,300 pounds.

B X

92..[Refer to Figure 4-29.] What is the change of total drag for a 140,000-pound

Page 56: 1500 questions Very Important.pdf

airplane when configuration is changed from flaps 30o, gear down, to flaps 0o,

gear up, at a constant airspeed of 160 knots?

A.13,500 pounds.

B.13,300 pounds.

C.15,300 pounds.

A X

93..[Refer to Figure 4-29.] What is the maximum charted indicated airspeed

while maintaining a 3o glide slope at a weight of 140,000 pounds?

A.127 knots.

B.149 knots.

C.156 knots.

C X

94..[Refer to Figure 4-29.] What is the thrust required to maintain a 3o glide

slope at 140,000 pounds, with gear down, flaps 30o, and an airspeed of VREF

+ 30 knots?

A.13,700 pounds.

B.16,200 pounds.

C.17,700 pounds.

B X

95..[Refer to Figure 4-29.] What thrust is required to maintain level flight at

140,000 pounds, with gear up, flaps 25o, and an airspeed of 172 knots?

A.13,700 pounds.

B.18,600 pounds.

C.22,000 pounds.

B X

96..[Refer to Figure 4-30.] What is the maximum charted indicated airspeed

while maintaining a 3o glide slope at a weight of 110,000 pounds?

A.136 knots.

B.132 knots.

C.139 knots.

A X

97..[Refer to Figure 4-30.] What thrust is required to maintain a 3o glide slope at

110,000 pounds, with gear down, flaps 30o, and an airspeed of VREF + 20

knots?

A.9,800 pounds.

B.11,200 pounds.

C.17,000 pounds.

B X

98..[Refer to Figure 4-30.] What thrust is required to maintain level flight at

110,000 pounds, with gear down, flaps 40o, and an airspeed of 118 knots?

A.17,000 pounds.

B.20,800 pounds.

C.22,300 pounds.

B X

Page 57: 1500 questions Very Important.pdf

99..[Refer to Figure 4-32.] What is the turbulent air penetration N1 power

setting for Operating Conditions below? WEIGHT (*1000): 100,PRESSURE

ALTITUDE: 35,000,TOTAL AIR TEMP (TAT): -11oC

A.83.4 percent.

B.86.0 percent.

C.87.4 percent.

C X

100..[Refer to Figure 4-32.] What is the turbulent air penetration N1 power

setting for Operating Conditions below? WEIGHT (*1000): 80,PRESSURE

ALTITUDE: 25,000,TOTAL AIR TEMP (TAT): -28oC

A.77.5 percent.

B.75.3 percent.

C.79.4 percent.

B X

101.[Refer to Figure 4-32.] What is the turbulent air penetration N1 power

setting for Operating Conditions below? WEIGHT (*1000): 90,PRESSURE

ALTITUDE: 15,000,TOTAL AIR TEMP (TAT): 11oC,

A.75.4 percent.

B.74.2 percent.

C.72.9 percent.

A X

102..[Refer to Figure 4-34.] What are the recommended IAS and EPR settings

for holding under Operating Conditions below? ,ALTITUDE: 30,000,WEIGHT

(*1000): 92.5,ENGINES OPERATING: 2,HOLDING TIME (MIN): 10

A.221 knots and 1.83 EPR.

B.210 knots and 1.69 EPR.

C.217 knots and 1.81 EPR.

B X

103..[Refer to Figure 4-34.] What are the recommended IAS and EPR settings

for holding under Operating Conditions below? ALTITUDE: 25,000,WEIGHT

(*1000): 102.5,ENGINES OPERATING: 2,HOLDING TIME (MIN): 40

A.216 knots and 1.62 EPR.

B.223 knots and 2.01 EPR.

C.219 knots and 1.81 EPR.

A X

104..[Refer to Figure 4-34.] What are the recommended IAS and EPR settings

for holding under Operating Conditions below? ALTITUDE: 35,000,WEIGHT

(*1000): 100,ENGINES OPERATING: 2,HOLDING TIME (MIN): 30

A.221 knots and 1.83 EPR.

B.223 knots and 2.01 EPR.

C.217 knots and 1.95 EPR.

C X

105..[Refer to Figure 4-34.] What is the approximate fuel consumed when

holding under Operating Conditions below? ,ALTITUDE: 35,000,WEIGHT

Page 58: 1500 questions Very Important.pdf

(*1000): 100,ENGINES OPERATING: 2,HOLDING TIME (MIN): 30

A.1,625 pounds.

B.1,950 pounds.

C.2,460 pounds.

C X

106..[Refer to Figure 4-34.] What is the approximate fuel consumed when

holding under Operating Conditions below? ALTITUDE: 25,000,WEIGHT

(*1000): 102.5,ENGINES OPERATING: 2,HOLDING TIME (MIN): 40

A.3,625 pounds.

B.3,240 pounds.

C.2,980 pounds.

B X

107..[Refer to Figure 4-34.] What is the approximate fuel consumed when

holding under Operating Conditions below? ALTITUDE: 30,000,WEIGHT

(*1000): 92.5,ENGINES OPERATING: 2,HOLDING TIME (MIN): 10

A.732 pounds.

B.1,023 pounds.

C.1,440 pounds.

A X

108..[Refer to Figure 4-37.] How many minutes of dump time is required to

reach a weight of 144,500 pounds? Initial weight: 180,500 lb,Zero fuel weight:

125,500 lb

A.13 minutes.

B.15 minutes.

C.16 minutes.

B X

109..[Refer to Figure 4-37.] How many minutes of dump time is required to

reach a weight of 151,500 pounds? Initial weight: 181,500 lb,Zero fuel weight:

126,000 lb

A.15 minutes.

B.14 minutes.

C.13 minutes.

C X

110..[Refer to Figure 4-37.] How many minutes of dump time is required to

reach a weight of 151,500 pounds? Initial weight: 187,500 lb,Zero fuel weight:

125,500 lb

A.14 minutes.

B.16 minutes.

C.17 minutes.

C X

111..[Refer to Figure 4-37.] How many minutes of dump time is required to

reduce fuel load to 18,000 pounds? Initial weight: 162,500 lb,Zero fuel weight:

120,500 lb

A.10 minutes.

Page 59: 1500 questions Very Important.pdf

B.9 minutes.

C.8 minutes.

A X

112..[Refer to Figure 4-39.] What is the approximate level-off pressure altitude

after drift-down under Operating Conditions below? WT AT ENG FAIL (*1000):

100,ENGINE ANTI-ICE: ON,WING ANTI-ICE: ON,ISA TEMPERATURE:

+10oC,AIR CONDITIONING: ON

A.16,400 feet.

B.19,600 feet.

C.18,700 feet.

A X

113..[Refer to Figure 4-39.] What is the approximate level-off pressure altitude

after drift-down under Operating Conditions below? WT AT ENG FAIL (*1000):

80,ENGINE ANTI-ICE: ON,WING ANTI-ICE: OFF,ISA TEMPERATURE:

ISA,AIR CONDITIONING: OFF

A.24,600 feet.

B.23,400 feet.

C.20,000 feet.

A X

114..[Refer to Figure 4-39.] What is the approximate level-off pressure altitude

after drift-down under Operating Conditions below?WT AT ENG FAIL (*1000):

90,ENGINE ANTI-ICE: OFF,WING ANTI-ICE: OFF,ISA TEMPERATURE:

+20oC,AIR CONDITIONING: OFF

A.19,400 feet.

B.20,000 feet.

C.23,800 feet.

B X

115..[Refer to Figure 4-41.] What is the approximate landing weight for

Operating Conditions below? WEIGHT (START TO ALT): 77,000,DISTANCE

(NM): 70,WIND COMPONENT: 25TW,HOLDING TIME AT ALT (MIN): 15

A.74,000 pounds.

B.74,180 pounds.

C.73,400 pounds.

A X

116..[Refer to Figure 4-41.] What is the approximate landing weight for

Operating Conditions below? WEIGHT (START TO ALT): 87,000,DISTANCE

(NM): 370,WIND COMPONENT: 60HW,HOLDING TIME AT ALT (MIN): 15

A.80,850 pounds.

B.85,700 pounds.

C.77,600 pounds.

A X

117..[Refer to Figure 4-41.] What is the total time from starting to the alternate

through completing the approach for Operating Conditions below? WEIGHT

(START TO ALT): 71,000,DISTANCE (NM): 200,WIND COMPONENT:

Page 60: 1500 questions Very Important.pdf

30TW,HOLDING TIME AT ALT (MIN): 15

A.34 minutes.

B.55 minutes.

C.57 minutes.

C X

118..[Refer to Figure 4-41.] What is the total time from starting to the alternate

through completing the approach for Operating Conditions below? WEIGHT

(START TO ALT): 84,000,DISTANCE (NM): 120,WIND COMPONENT:

20HW,HOLDING TIME AT ALT (MIN): 15

A.30 minutes.

B.45 minutes.

C.29 minutes.

B X

119..[Refer to Figure 4-43.] What is the trip fuel for Operating Conditions below?

DISTANCE (NM): 2200,WIND COMPONENT (KTS): 50HW,CRUISE PRESS

ALTITUDE: 37,000,ISA TEMPERATURE: +10oC,LANDING WEIGHT (*1000):

75

A.34,000 pounds.

B.28,000 pounds.

C.32,600 pounds.

C X

120..[Refer to Figure 4-43.] What is the trip fuel for Operating Conditions below?

DISTANCE (NM): 2500,WIND COMPONENT (KTS): 50TW,CRUISE PRESS

ALTITUDE: 27,000,ISA TEMPERATURE: ISA,LANDING WEIGHT (*1000): 70

A.35,000 pounds.

B.32,600 pounds.

C.30,200 pounds.

B X

121..[Refer to Figure 4-43.] What is the trip time for Operating Conditions below?

DISTANCE (NM): 2200,WIND COMPONENT (KTS): 50HW,CRUISE PRESS

ALTITUDE: 37,000,ISA TEMPERATURE: +10oC,LANDING WEIGHT (*1000):

75

A.5 hours 15 minutes.

B.5 hours 40 minutes.

C.5 hours 55 minutes.

B X

122..[Refer to Figure 4-43.] What is the trip time for Operating Conditions below?

DISTANCE (NM): 2500,WIND COMPONENT (KTS): 50TW,CRUISE PRESS

ALTITUDE: 27,000,ISA TEMPERATURE: ISA,LANDING WEIGHT (*1000): 70

A.5 hours 3 minutes.

B.5 hours 20 minutes.

C.5 hours 55 minutes.

A X

123..[Refer to Figure 4-45.] What is the estimated fuel consumption for

Page 61: 1500 questions Very Important.pdf

Operating Conditions below? DISTANCE (NM): 600,AVG WIND COMP (KTS):

40HW

A.9,926 pounds.

B.9,680 pounds.

C.9,504 pounds.

C X

124..[Refer to Figure 4-45.] What is the estimated fuel consumption for

Operating Conditions below? DISTANCE (NM): 750,AVG WIND COMP (KTS):

30HW

A.11,227 pounds.

B.11,503 pounds.

C.11,754 pounds.

A X

125..[Refer to Figure 4-45.] What is the trip time corrected for wind under

Operating Conditions below? DISTANCE (NM): 400,AVG WIND COMP (KTS):

15TW,

A.58.1 minutes.

B.59.9 minutes.

C.54.7 minutes.

B X

126..[Refer to Figure 4-45.] What is the trip time corrected for wind under

Operating Conditions below? DISTANCE (NM): 850,AVG WIND COMP (KTS):

65TW,

A.97.2 minutes.

B.99.8 minutes.

C.103.7 minutes.

C X

127..[Refer to Figures 4-22 and 4-24.] What is the go-around EPR for Operating

Conditions L-1?

A.2.01 EPR.

B.2.03 EPR.

C.2.04 EPR.

B X

128..[Refer to Figures 4-22 and 4-24.] What is the go-around EPR for Operating

Conditions L-2?

A.2.115 EPR.

B.2.10 EPR.

C.2.06 EPR.

C X

129..[Refer to Figures 4-22, 4-23 and 4-24.] What is the maneuvering speed for

Operating Conditions L-5?

A.124 knots.

B.137 knots.

C.130 knots.

Page 62: 1500 questions Very Important.pdf

C X

130..[Refer to Figures 4-22, 4-23 and 4-24.] What is the reference speed for

Operating Conditions L-2?

A.140 knots.

B.145 knots.

C.148 knots.

B X

131..[Refer to Figures 4-22, 4-23 and 4-24.] What is VREF for Operating

Conditions L-1?

A.143 knots.

B.144 knots.

C.145 knots.

A X

132..[Refer to Figures 4-22, and 4-24.] What is the go-around EPR for

Operating Conditions L-3?

A.2.06 EPR.

B.2.07 EPR.

C.2.09 EPR.

A X

133..[Refer to Figures 4-25 and 4-26.] Which conditions will result in the

shortest landing distance at a weight of 132,500 pounds?

A.Dry runway using brakes and reversers.

B.Dry runway using brakes and spoilers.

C.Wet runway using brakes, spoilers and reversers.

B X

134..[Refer to Figures 4-28 and 4-29.] What approach speed and ground roll

will be needed when landing at a weight of 140,000 pounds if flaps are not

used?

A.138 knots and 3,900 feet.

B.153 knots and 2,900 feet.

C.183 knots and 2,900 feet.

C X

135..[Refer to Figures 4-35 and 4-36.] What are descent fuel and distance

under Operating Conditions S-4?

A.1,490 pounds, 118 NAM.

B.1,400 pounds, 110 NAM.

C.1,430 pounds, 113 NAM.

C X

136..[Refer to Figures 4-35 and 4-36.] What are descent fuel and distance

under Operating Conditions S-5?

A.1,420 pounds, 97 NAM.

B.1,440 pounds, 102 NAM.

C.1,390 pounds, 92 NAM.

A X

Page 63: 1500 questions Very Important.pdf

137..[Refer to Figures 4-35 and 4-36.] What are descent time and distance

under Operating Conditions S-1?

A.24 minutes, 118 NAM.

B.26 minutes, 125 NAM.

C.25 minutes, 118 NAM.

B X

138..[Refer to Figures 4-35 and 4-36.] What are descent time and distance

under Operating Conditions S-2?

A.24 minutes, 109 NAM.

B.25 minutes, 125 NAM.

C.23 minutes, 118 NAM.

C X

139..[Refer to Figures 4-46, 4-47 and 4-48.] What are the fuel requirements

from Chicago Midway Airport to Greater Buffalo Intl?

A.2,224 pounds.

B.1,987 pounds.

C.1,454 pounds.

A X

140..[Refer to Figures 4-49, 4-50,4-51,4-52,4-53, and 4-54.] What is the ETE

at .78 Mach?

A.1 hours 08 minutes.

B.1 hours 02 minutes.

C.1 hours 05 minutes.

A X

141..[Refer to Figures 4-49, 4-50,4-51,4-52,4-53, and 4-54.] What is the ETE

at .80 Mach?

A.1 hours 02 minutes.

B.1 hours 04 minutes.

C.1 hours 07 minutes.

C X

142..[Refer to Figures 4-50,4-51,4-52,4-53, and 4-54.] What is the total fuel

required at .78 Mach?

A.22,140 pounds.

B.22,556 pounds.

C.22,972 pounds.

B X

143..[Refer to Figures 4-50,4-51,4-52,4-53, and 4-54.] What is the total fuel

required at .80 Mach?

A.22,836 pounds.

B.22,420 pounds.

C.22,556 pounds.

B X

144..A commercial operator plans to ferry a large, four-engine,

reciprocating-engine-powered airplane from one facility to another to repair an

Page 64: 1500 questions Very Important.pdf

inoperative engine. Which is an operational requirement for the three-engine

flight?

A.The gross weight at takeoff may not exceed 75 percent of the maximum

certificated gross weight.

B.Weather conditions at the takeoff and destination airports must be VFR.

C.The computed takeoff distance to reach V1 must not exceed 70 percent of

the effective runway length.

B X

145..A definition of the term "viscous hydroplaning" is where

A.the airplane rides on standing water.

B.a film of moisture covers the painted or rubber-coated portion of the runway.

C.the tires of the airplane are actually riding on a mixture of steam and melted

rubber.

B X

146..An outside air pressure decreases, thrust output will

A.increase due to greater efficiency of jet aircraft in thin air.

B.remain the same since compression of inlet air will compensate for any

decrease in air pressure.

C.decrease due to higher density altitude.

C X

147..At what minimum speed (rounded off) could dynamic hydroplaning occur

on main tires having a pressure of 100 PSI?

A.90 knots.

B.96 knots.

C.110 knots.

A X

148..At what minimum speed (rounded off) could dynamic hydroplaning occur

on main tires having a pressure of 110 PSI?

A.90 knots.

B.95 knots.

C.100 knots.

B X

149..At what minimum speed (rounded off) could dynamic hydroplaning occur

on main tires having a pressure of 121 PSI?

A.90 knots.

B.96 knots.

C.110 knots.

B X

150..At what speed, with reference to L/DMAX, does maximum range for a jet

airplane occur?

A.A speed less than that for L/DMAX.

B.A speed equal to that for L/DMAX.

C.A speed greater than that for L/DMAX.

C X

Page 65: 1500 questions Very Important.pdf

151..At what speed, with reference to L/DMAX, does maximum rate-of-climb for

a jet airplane occur?

A.A speed greater than that for L/DMAX.

B.A speed equal to that for L/DMAX.

C.A speed less than that for L/DMAX.

B X

152..Compared to dynamic hydroplaning, at what speed does viscous

hydroplaning occur when landing on a smooth, wet runway?

A.At approximately 2.0 times the speed that dynamic hydroplaning occurs.

B.At a lower speed than dynamic hydroplaning.

C.At the same speed as dynamic hydroplaning.

B X

153..Equivalent shaft horsepower (ESHP) of a turboprop engine is a measure

of

A.turbine inlet temperature.

B.shaft horsepower and jet thrust.

C.propeller thrust only.

B X

154..How should reverse thrust propellers be used during landing for maximum

effectiveness in stopping?

A.Gradually increase reverse power to maximum as rollout speed decreases.

B.Use maximum reverse power as soon as possible after touchdown.

C.Select reverse-pitch after landing and use idle power setting of the engines.

B X

155..How should thrust reversers be applied to reduce landing distance for

trubojet aircraft?

A.Immediately after ground contact.

B.Immediately prior to touchdown.

C.After applying maximum wheel braking.

A X

156..If an engine failure occurs at an altitude above single-engine ceiling, what

airspeed should be maintained?

A.VMC.

B.VYSE.

C.VXSE.

B X

157..In a light ,twin-engine airplane with one engine inoperative, when is it

acceptable to allow the ball of a slip-skid indicator to be deflected outside the

reference lines?

A.-While maneuvering at minimum controllable air-speed to avoid over banking.

B.-When operating at any airspeed greater than Vmc.

C.-When practicing imminent stalls in a banked attitude.

B X

Page 66: 1500 questions Very Important.pdf

5.重量与平衡重量与平衡重量与平衡重量与平衡

1..What are some characteristics of an airplane loaded with the CG at the aft

limit?

A.Lowest stall speed, highest cruise speed, and least stability

B.Highest stall speed, highest cruise speed, and least stability.

C.Lowest stall speed, lowest cruise speed, and highest stability.

A X

2..What characteristic should exist if an airplane is loaded to the rear of its CG

range?

A.Sluggish in aileron control.

B.Sluggish in rudder control

C.Unstable about the lateral axis

C X

3..As the CG is moved forward, the stability

A.Improves

B.Decrease

C.dose not change

A X

4..In flight ,as the time goes on , the CG will

A.change

B.not change

C.may change or may not change

A X

5..In small airplanes, normal recovery form spins may become difficult if the

A.-CG is too far rearward and rotation is around the longitudinal axis.

B.-CG is too far rearward and rotation is around the CG.

C.-spin is entered before the stall is fully developed.

C X

6.航班运行航班运行航班运行航班运行

1..Information recorded during normal operation of a cockpit voice recorder in a

large pressurized airplane with four reciprocating engines

A.May all be erased or otherwise obliterated except for the last 30 minutes

B.May be erased or otherwise obliterated except for the last 30 minutes prior to

landing

C.May all be erased, as the voice recorder is not required on an aircraft with

reciprocating engines.

A X

2..Information obtained from flight data and cockpit voice recorders shall be

used only for determining

A.Who was responsible for any accident or incident

B.Evidence for use in civil penalty or certificate action

C.Possible causes of accidents or incidents

C X

Page 67: 1500 questions Very Important.pdf

3..Land and Hold Short Operations (LAHSO) include landing and holding short:

A.Of an intersecting taxiway only.

B.Of some designated point on the runway.

C.Only of an intersecting runway or taxiway.

B X

4..Normally, a dispatcher should be scheduled for no more than

A.8 hours of service in any 24 consecutive hours

B.10 hours of duty in any 24 consecutive hours

C.10 consecutive hours of duty

C X

5..Northwest wing can be presented as .

A. 200°or NW

B. 315°or NW

C. 135°or SE

B X

6..Prior to listing an aircraft as an alternate airport in the dispatch release,

weather reports must indicate that weather conditions will be at or above

authorized minimums at that airport

A.during the entire flight.

B.for the period of 1 hour before to 1 hour after the ETA.

C.when the flight arrives.

C X

7..Regulations concerning the operational control of a flight refer to

A.the specific duties of any required crewmember.

B.exercising authority over initiating, conducting, or terminating a flight.

C.exercising the privileges of pilot-in-command of an aircraft.

B X

8..Route that require a flight navigator are listed in the

A.Airplane Flight Manual

B.International Flight Information Manual

C.Air Carrier's Operations Specifications

C X

9..Series A NOTAMs is mainly released to

A.domestic airport flight information office.

B.foreign international NOTAMs office.

C.domestic area flight information center.

B X

10..Series C NOTAMs of Class 2 must be disseminated to domestic area flight

information center NOTAMs office

A.25 days ago.

B.15 days ago.

C.7 days ago.

B X

11..The "runway hold position" sign denotes

Page 68: 1500 questions Very Important.pdf

A.An area protected for an aircraft approaching a runway

B.An entrance to runway from a taxiway

C.Intersecting runways

C X

12..The ATS unit must ensure that pilots are kept informed of any change in the

status of airport facilities_______________.

A. at their destinations

B. at their alternate aerodromes

C. A and B

C X

What terms could a ATC furnishes pilots' braking action reports using?

A. "good", "fair", "poor" and "nil" or a combination of these terms.

B. the percentage of the proximate braking action.

C. only use "good", "fair", "poor" and "nil" with no combination of these terms.

A X

When a speed adjustment is necessary to maintain separation, what minimum

speed may ATC request of an aircraft operating above 3,000 meters, which

normal cruising speed is 150 knots?

A. 150 knots

B. 250 knots

C. 330 knots

A X

when a speed adjustment is necessary to maintain separation, what minimum

speed may ATC request of a piston aircraft departing an airport?

A. 170 knots

B. 150 knots

C. 130 knots

B X

When a speed adjustment is necessary to maintain separation, what minimum

speed may ATC request of a piston arrival aircraft operating except 20 miles

from threshold?

A. 200 knots

B. 170 knots

C. 150 knots

C X

When a speed adjustment is necessary to maintain separation, what minimum

speed may ATC request of a turbojet arrival aircraft operating except 20 miles

from threshold?

A. 200 knots

B. 170 knots

C. 150 knots

B X

When a speed adjustment is necessary to maintain separation, what minimum

speed may ATC request of a turboprop arrival aircraft operating except 20 miles

Page 69: 1500 questions Very Important.pdf

from threshold?

A. 200 knots

B. 170 knots

C. 150 knots

C X

When a speed adjustment is necessary to maintain separation, what minimum

speed may ATC request of a piston arrival aircraft operating below 3,000

meters?

A. 210 knots

B. 200 knots

C. 150 knots

B X

When a speed adjustment is necessary to maintain separation, what minimum

speed may ATC request of a turboprop arrival aircraft operating below 3,000

meters?

A. 210 knots

B. 200 knots

C. 170 knots

B X

when a speed adjustment is necessary to maintain separation, what minimum

speed may ATC request of a turbojet aircraft departing an airport?

A. 250 knots

B. 230 knots

C. 210 knots

B X

What is the hijack code?

A. 7200.

B. 7500.

C. 7777.

B X

The fuel reserve required for a reciprocating-engine-powered supplemental air

carrier airplane upon arrival at the most distant alternate airport during a flight in

the contiguous China is

A. 45 minutes at normal cruising fuel consumption.

B. The fuel required to fly to the alternate, plus 10 percent.

C. 3 hours at normal cruising fuel consumption.

A X

The minimum amount (planned) of fuel to be aboard a flag carrier turbojet

airplane on a flight within the contiguous China , after reaching the most distant

alternate airport, should be

A. 45 minutes at normal cruising fuel consumption.

B. 2 hours at normal cruising fuel consumption.

C. enough fuel to return to the destination airport or to fly for 90 minutes at

normal cruising fuel consumption, whichever is less.

Page 70: 1500 questions Very Important.pdf

A X

Upon arriving at the most distant airport, what is the fuel reserve requirement

for a turbopropeller flag air carrier airplane?

A. 90 minutes at holding altitude and speed fuel consumption or 30 minutes

plus 15 percent of cruise fuel consumption, whichever is less.

B. 45 minutes at holding altitude.

C. 30 minutes plus 15 percent of the total time required, or 90 minutes at normal

cruise, whichever is less.

C X

What phases is the Critical Flight Phases?

A. climb and descent when below 3,000 meters, taxi, takeoff, and landing.

B. climb and descent when below 3,000 meters.

C. takeoff, and landing.

A X

What action is necessary when a partial loss of ILS receiver capability occurs

while operating in controlled airspace under IFR?

A. Continue as cleared and file a written report to the Administrator if requested.

B. If the aircraft is equipped with other radios suitable for executing an

instrument approach, no further action is necessary

C. Report the malfunction immediately to ATC.

C X

What altitude and route should be used if the pilot is flying in IFR weather

conditions and has two-way radio communications failure?

A. Continue on the route specified in the clearance and fly the highest of the

following: the last assigned altitude, altitude ATC has informed the pilot to

expect, or to the MEA.

B. Descend to MEA and, if clear of clouds, proceed to the nearest appropriate

airport. If not clear of clouds, maintain the highest of the MEAs along the

clearance route.

C. Fly the most direct route to the destination, maintaining the last assigned

altitude or MEA, whichever is higher.

A X

Instructions are directives issued by ATC for the purpose of

A.requiring a pilot to read back.

B.keeping flight status.

C.requiring a pilot to take specific action.

C X

It should be reported without ATC request that

A. When unable to climb at a rate of at least 500 feet per minute.

B. When unable to descend at a rate of at least 1000 feet per minute.

C. When unable to climb or descend at a rate of at least 600 feet per minute.

A X

The bottom of the Class B airspace is appropriate to

A.FL 6,300m.

Page 71: 1500 questions Very Important.pdf

B.FL 6,000m.

C.FL 600m.

C X

The cruising speed of an aircraft is 800 kilometers, what should be entered in

the CRUISING SPEED section of the flight plan?

A.850.

B.K0850.

C.0850K.

B X

The lowest ILS Category II minimums are

A.DH 50 feet and RVR 1,200 feet.

B.DH 100 feet and RVR 1,200 feet.

C.DH 150 feet and RVR 1,500 feet.

B X

The propose of the speed adjustment issued by ATC to the radar-controlled

aircraft are?

A. to achieve the desired separation.

B. to maintain enough separation;

C. both a and b

C X

The top of Class D airspace corresponds to

A.600m AGL.

B.Final approach fix.

C.The first holding pattern level.

C X

Under what condition does ATC issue safety alerts?

A. When collision with another aircraft is imminent.

B. If the aircraft altitude is noted to be in close proximity to the surface or an

obstacle.

C. When weather conditions are extreme and wind shear or large hail is in the

vicinity.

B X

Under what condition should a pilot on IFR advise ATC of minimum fuel status?

A. When the fuel supply becomes less than that required for IFR.

B. If the remaining fuel suggests a need for traffic or landing priority.

C. If the remaining fuel precludes any undue delay.

C X

What does the term "minimum fuel" imply to ATC?

A. Traffic priority is needed to the destination airport.

B. Emergency handling is required to the nearest suitable airport.

C. Advisory that indicates an emergency situation is possible should an undue

delay occur.

C X

What is the lowest HAT for which a Category II applicant can be certified during

Page 72: 1500 questions Very Important.pdf

the original issuance of the authorization?

A.100 feet AGL.

B.150 feet AGL.

C.200 feet AGL.

B X

What is the maximum distance that a departure alternate airport may be from

the departure airport for a two-engine airplane?

A.2 hours at normal cruising speed in still air with one engine operating.

B.1 hour at normal cruising speed in still air with one engine inoperative.

C.1 hour at normal cruising speed in still air with both engines operating.

B X

What is the purpose of a prohibited airspace?

A.To prohibit entry of any aircrafts.

B.To prohibit entry of air force aircrafts only.

C.To prohibit entry of civil aircrafts only.

A X

What is the purpose of an ATC instruction?

A.To require a pilot to take a specific action.

B.To state some important information.

C.To warn the pilot.

A X

What is the purpose of the danger airspace?

A.To warn nonparticipating aircrafts of the potential danger.

B.To prohibit nonparticipating aircrafts of entry.

C.To protect military activities.

A X

What is the time interval for filing flight plan?

A.At least one hour prior to the departure time.

B.At least 30 minutes prior to the departure time.

C.At least one hour and a half prior to the departure time.

C X

What is the time limitation for filing flight application?

A.At least before 1500 Beijing Time prior to the departure date.

B.At least before 1700 Beijing Time prior to the departure date.

C.At least before 1200 Beijing Time prior to the departure date.

A X

What report should the pilot make at a clearance limit?

A. time and altitude/flight level arriving or leaving.

B. time, altitude/flight level, and expected holding speed.

C. time, altitude/flight level, expected holding speed, and inbound leg length.

A X

When a flight plan indicates IFR for the entire route, pilot should fill the FLIGHT

RULES box with letter

A.Y.

Page 73: 1500 questions Very Important.pdf

B.I.

C.Z.

B X

When a pilot plans a flight using NDB NAVAIDS. Which rule applies?

A.The airplane must have sufficient fuel to proceed. by means of VOR NAVIDS,

to a suitable airport and land.

B.The pilot must be able to return to the departure airport using other navigation

radios.

C.The airplane must have sufficient fuel to proceed, by means of VOR

NAVAIDS, to a suitable airport and complete an instrument approach by use of

the remaining airplane radio system.

C X

When an air carrier flight is operated under IFR or over-the-top on "victor

airways," which navigation equipment is required to be installed in duplicate?

A.VOR

B.ADF

C.VOR and DME

A X

When an ATC controller assigns a too low speed for the aircraft's operating

limitation under the existing circumstance, what should the pilot do?

A. reduce speed to the assigned speed.

B. reduce speed to the minimum speed at that circumstance.

C. advise ATC of the speed that will be used.

C X

When can a pilot expect landing priority?

A. on a first-come, first serve basis, because the Air Traffic Control towers

handle all aircraft regardless of the type of flight plan.

B. during emergency

C. both a and b

C X

When can an aircraft be expected of changing frequency from tower to ground

control after landing?

A. just after landing and stop taxiing on the runway.

B. remain on the tower frequency until instructed to do otherwise.

C. when exit the runway.

B X

When carrying a passenger aboard an all-cargo aircraft, which of the following

applies?

A. The passenger must have access to a seat in the pilot compartment.

B. The pilot in command may authorize the passenger to be admitted to the

crew compartment.

C. Crew-type oxygen must be provided for the passenger.

B X

When proceeding to the alternate airport, which minimums apply?

Page 74: 1500 questions Very Important.pdf

A.The alternate minimums calculated from the IAP chart.

B.The actual minimums shown on the IAP chart.

C.visibility 1,600m and ceiling 120m.

B X

When the pilot in command is responsible for a deviation during an emergency,

the pilot should submit a written report

A. within 10 days after the deviation.

B. after returning home.

C. after the flight is completed.

C X

When the tower controller instructs the taxiway to the aircraft during landing taxi,

which taxi way can an aircraft use?

A. exit the runway at the nearest suitable taxiway without delay.

B. exit the runway at the taxiway given by the tower controller.

C. exit the runway at the suitable taxiway on the pilot's determination.

B X

When vectoring to intercept the ILS localizer course or MLS final approach

track, the final vector shall be such as to enable the aircraft to intercept the ILS

localizer course or MLS final approach track at an angle not greater

A.(续正文 than _____and to provide at least _____straight and level flight prior

to ILS localizer course or MLS final approach track intercept.) 30 degrees, 2 km

B. 40 degrees, 2 km

C. 30 degrees, 1.5 km

A X

Where does a danger airspace may be located?

A.Over the territory only.

B.Over the territory or open sea abut to China.

C.Over the open sea only.

B X

Where does a prohibited airspace be set up?

A.Over domestic only.

B.Over open sea.

C.Over domestic or open sea.

A X

Where does a restricted airspace be set up?

A.Over open sea.

B.Over domestic only.

C.Over domestic and open sea.

B X

Which altitude is appropriate for the ceiling of Class C airspace?

A.FL 6,000m.

B.FL 6,300m.

C.FL 6,600m.

A X

Page 75: 1500 questions Very Important.pdf

Which altitude is appropriate for the floor of Class A airspace?

A.FL 6,000m.

B.FL 6,300m.

C.The first holding pattern altitude.

B X

Which altitude is appropriate to the top of Class B airspace?

A.FL 6,300m.

B.FL 6,000m.

C.FL 600m.

B X

Which letter do you select to fill in the TYPE OF FLIGHT box during IFR flight

training?

A.N.

B.S.

C.G.

C X

Which of following might not normally need be reported without ATC request?

A. expected holding speed

B. vacating a previously assigned altitude for a newly assigned one.

C. when unable to climb or descend at a rate of at least 500 feet per minute.

A X

Which of following should be reported without ATC request as a compulsory

report?

A. passengers on board

B. ordering galley supplies

C. when an approach has been missed.

C X

Which points should be report to ATC without request?

A. when leaving the final approach fix outbound.

B. when leaving an assigned holding fix.

C. when leaving the final approach fix outbound, or when leaving an assigned

holding fix.

C X

Which procedure produces the minimum fuel consumption for a given leg of the

cruise flight?

A.Increase speed for a headwind.

B.Increase speed for a tailwind.

C.Increase altitude for a headwind, decrease altitude for a tailwind.

A X

Which regulation does the pilot must comply with during operation within Class

A airspace?

A.Approach Control Regulation.

B.Visual Flight Regulation.

C.Instrument Control Regulation.

Page 76: 1500 questions Very Important.pdf

C X

Which reports are required when operating IFR in radar environment?

A. Position reports, vacating an altitude, unable to climb 500 ft/min, and time

and altitude reaching a holding fix or point to which cleared.

B. Position reports, vacating an altitude, unable to climb 500 ft/min, and time

and altitude reaching a holding fix or point to which cleared, and a change in

average true airspeed exceeding 5 percent or 10 knots.

C. Vacating an altitude, unable to climb 500 ft/min, reaching a holding fix or

point to which cleared, a change in average true airspeed exceeding 5 percent

or 10 knots, and leaving any assigned holding fix or point.

C X

While flying IFR in controlled airspace, if one of the two VOR receivers fails,

which course of action should the pilot-in-command follow?

A. No call is required if one of the two VOR receivers is operating properly.

B. Advise ATC immediately.

C. Notify the dispatcher via company frequency.

B X

While in IFR conditions, a pilot experiences two-way radio communications

failure. Which route should be flown in the absence

A.(续正文,of an ATC assigned route or a route ATC has advised to expect in a

further clearance?) The most direct route to the filed alternate airport.

B. An off-airway route to the point of departure.

C. The route filed in the flight plan.

C X

Who is responsible for the issue of Class 2 NOTAMs?

A.Domestic flight information center NOTAMs office.

B.ATMB NOTAMs office.

C.International NOTAMs office of CAAC flight information center.

C X

Who is responsible for the issue of Series D NOTAMs?

A.CCAFC.

B.Area flight information center.

C.Airport flight information office.

C X

Who is responsible for the issue of SNOWTAM?

A.ATMB NOTAMs office.

B.Area flight information center.

C.Airport flight information office.

C X

Who is responsible of arranging the separation of the aircrafts within Class A

airspace?

A.Air Traffic Controller.

B.Pilot-in-command.

C.Dispatcher.

Page 77: 1500 questions Very Important.pdf

A X

Why the ATC ask the aircraft for speed adjustment?

A. to check the performance of the aircraft;

B. because ATC do think high speed is not good for safety;

C. to achieve or maintain the desired separation.

C X

You can't enter tower controlled airspace under IFR weather conditions unless

you have filed an IFR flight plan and

A.slowed down to the final approach IAS.

B.received an ATC clearance.

C.climbed to the appropriate altitude.

B X

You have just landed at JFK and the tower tells you to call ground control when

clear of the runway. You are considered clear of runway when

A.The aft end of the aircraft is even with the taxiway location sign

B.The flight deck area of the aircraft is even with the hold line

C.All parts of aircraft have crossed the hold line.

C X

You should advise ATC of minimum fuel status when your fuel supply has

reached a state where, upon reaching your destination, you cannot accept any

undue delay.

A. This will ensure your priority handling by ATC.

B. ATC will consider this action as if you had declared an emergency.

C. If your remaining usable fuel supply suggests the need for traffic priority to

ensure a safe landing, declare an emergency due to low fuel and report fuel

remaining in minutes.

C X

Who is required to submit a written report on a deviation that occurs during an

emergency?

A. Person who found the emergency.

B. Person who heard the emergency.

C. Pilot in command.

C X

Who is responsible for obtaining information on all current airport conditions,

weather, and irregularities of navigation facilities for a supplemental air carrier

flight?

A.Aircraft dispatcher.

B.Director of operations or flight follower.

C.Pilot in command.

C X

While on an IFR flight in controlled airspace, the failure of which unit will

precipitate an immediate report to ATC?

A.One engine, on a multiengine aircraft

B.Airborne radar

Page 78: 1500 questions Very Important.pdf

C.DME

C X

(1-1) At a waypoint , PTL 130 is notified that the Chongqing Airport is closed.

PTL 130 is told to proceed to Chengdu Inti Airport.PTL 130 is operating under

CCAR Part 121. The PIC on PTL 130 has less than 100 hours as PIC in the

B-727.

A.(续正文考题,What are PICs minimums for the VOR/DME RWY 02

approach)__MDH150m,VIS2400m.

B.MDH120m,VIS1600m.

C.MDH100m,VIS1600m.

B X

(Refer to Figure 6-1) At what point must the missed approach be initiated on the

VOR/DME Rwy 36R at CAPITAL, if still IMC?

A.D0.3 PEK.

B.Above PEK VOR.

C.D0.2 PEK.

B X

(Refer to Figure 6-1) During final approach, the altitude of LOM is 1,360 feet,

the height of LOM to the touchdown zone is

A.1,360 feet.

B.1,245 feet.

C.1,262 feet.

C X

(Refer to Figure 6-1) During the VOR/DME Rwy 36R approach while

maintaining an on glide slope indication with a groundspeed of 140 knots, what

was the appropriate rate of descent?

A.632 feet per minute.

B.843 feet per minute.

C.737 feet per minute.

C X

(Refer to Figure 6-1) How can the FAF on the VOR/DME Rwy 36R of CAPITAL

be identified?

A.11.7NM from PEK.

B.11.7KM from PEK.

C.11.7SM from PEK.

A X

(Refer to Figure 6-1) If the pilot has identified PEK on the VOR/DME Rwy 36R

straight-in land at CAPITAL, what height above the touchdown zone does a

Category C aircraft descend to in IMC?

A.372 feet.

B.470 feet.

C.525 feet.

A X

(Refer to Figure 6-1) The ATIS of CAPITAL airport is broadcast on the

Page 79: 1500 questions Very Important.pdf

frequency of

A.127.6MHz.

B.127.6KHz.

C.114.7MHz.

A X

(Refer to Figure 6-1) The FAF is indicated with a cross in the profile view of the

chart. To what height does the aircraft descend when it intercepts the final

approach segment?

A.1360 feet.

B.3940 feet.

C.3842 feet.

C X

(Refer to Figure 6-1) The highest terrain shown in the plan view section of the

VOR/DME Rwy 36R approach chart is

A.1,496 feet.

B.900 feet.

C.1,244 feet.

A X

(Refer to Figure 6-1) The highest terrain shown in the plan view section of the

VOR/DME Rwy 36R approach chart is 1,496 feet MSL, what is the height above

the airport of this obstruction then?

A.1,398 feet.

B.1,026 feet.

C.1,381 feet.

C X

(Refer to Figure 6-1) What are the landing minimums for B737 with ALS out?

A.MDA 470 feet and RVR 1,600m.

B.MDA 470 feet and VIS 1,500m.

C.MDA 470 feet and RVR 1,500m.

C X

(Refer to Figure 6-1) What is the frequency of the primary navaid facility on the

VOR/DME Rwy 36R approach at CAPITAL?

A.308KHz.

B.114.7MHz.

C.240KHz.

B X

(Refer to Figure 6-1) When the aircraft is intercepting R-188 HUR with an

intercept angle of 45o during go-around, how much is the intercept heading?

A.143o.

B.233o.

C.53o.

A X

(Refer to Figure 6-1) Which approach lighting is available for VOR/DME Rwy

36R?

Page 80: 1500 questions Very Important.pdf

A.HIALS.

B.HIALS with PAPI.

C.PAPI.

B X

(Refer to Figure 6-1) While being turning from D34.0 HUR to D16.0 PEK for the

VOR/DME Rwy 36R, Beijing Approach control tells pilot to contact the Tower,

what frequency should be tuned?

A.118.1KHz.

B.118.5KHz.

C.118.1MHz.

C X

(Refer to Figure 6-2) During the ILS/DME Rwy 36L approach while maintaining

an on glide slope indication with a groundspeed of 150 knots, what was the

approximate rate of descent?

A.760 feet per minute.

B.780 feet per minute.

C.800 feet per minute.

B X

(Refer to Figure 6-2) How can the FAF on the ILS/DME Rwy 36L of CAPITAL be

identified?

A.16.5KM from ILG.

B.16.5NM from ILG.

C.16.5SM from ILG.

B X

(Refer to Figure 6-2) How much is the height from LOM to the touchdown zone

while executing ILS/DME approach?

A.762 feet.

B.870 feet.

C.755 feet.

A X

(Refer to Figure 6-2) If an aircraft is cleared to circle-to-land on Rwy 36L at

CAPITAL, and its maximum speed is approximately 180 knots, what height

above the airport could it descend to?

A.870 feet.

B.762 feet.

C.755 feet.

C X

(Refer to Figure 6-2) If the pilot has identified IDK on the ILS/DME Rwy 36L

straight-in land at CAPITAL, what height above the airport does a Category B

aircraft descend to in IMC?

A.308 feet.

B.193 feet.

C.200 feet.

B X

Page 81: 1500 questions Very Important.pdf

(Refer to Figure 6-2) The aircraft has landed at 1100, pilot should contact with

Ground Control on the frequency of

A.121.7MHz.

B.121.9MHz.

C.121.7KHz.

B X

(Refer to Figure 6-2) The glide slope angle of ILS/DME Rwy 36L at CAPITAL is

3o, what is the corresponding rate of descent?

A.5.2%.

B.4.8%.

C.5.5%.

A X

(Refer to Figure 6-2) The highest terrain shown in the plan view section of the

ILS/DME Rwy 36L approach chart is

A.1,496 feet.

B.1,244 feet.

C.900 feet.

B X

(Refer to Figure 6-2) What is the frequency of the primary navaid facility on the

ILS/DME Rwy 36L approach at CAPITAL?

A.110.3MHz.

B.114.7MHz.

C.111.7MHz.

C X

(Refer to Figure 6-2) What is the identifier of HUAIROU VOR/DME?

A.HUR.

B.PEK.

C.IDK.

A X

(Refer to Figure 6-2) What is the procedure for initiating the missed approach

on the ILS/DME Rwy 36L at CAPITAL?

A.Climb straight ahead to L NDB, right turn, then select HUR VOR.

B.Climb straight ahead to 990 feet.

C.Climb straight ahead to LG NDB, right turn, then select HUR VOR.

C X

(Refer to Figure 6-2) Which approach lighting is available for ILS/DME Rwy

36L?

A.HIALS with PAPI.

B.HIALS.

C.PAPI.

A X

(Refer to Figure 6-3) If an aircraft has been aligned with the center line of Rwy

35 at PUDONG, the heading indicator should indicate

A.342o.

Page 82: 1500 questions Very Important.pdf

B.347o.

C.352o.

B X

(Refer to Figure 6-3) The magnetic direction of Rwy 17 at PUDONG is

A.162o.

B.172o.

C.167o.

C X

(Refer to Figure 6-3) The true direction of Rwy 35 at PUDONG is

A.342o.

B.347o.

C.352o.

A X

(Refer to Figure 6-3) What is the elevation of PUDONG airport?

A.8 feet.

B.10 feet.

C.12 feet.

B X

(Refer to Figure 6-3) What is the elevation of Rwy 17 at PUDONG?

A.8 feet.

B.12 feet.

C.10 feet.

C X

(Refer to Figure 6-3) What is the elevation of Rwy 35 at PUDONG?

A.10 feet.

B.8 feet.

C.12 feet.

A X

(Refer to Figure 6-3) Which frequency should be checked first while asking for

takeoff clearance at PUDONG airport?

A.124.35MHz.

B.127.85MHz.

C.118.8MHz.

C X

(Refer to Figure 6-3) Which frequency should be selected to check the airport

and weather conditions prior to departure from PUDONG?

A.127.85MHz.

B.124.35MHz.

C.121.65MHz.

A X

(Refer to Figure 6-3) While requesting the start-up time at PUDONG airport,

pilot should tune at first on the frequency of

A.124.35MHz.

B.121.65MHz.

Page 83: 1500 questions Very Important.pdf

C.127.85MHz.

B X

(Refer to Figure 6-3, 6-4) What distance is available for landing beyond on Rwy

35 at PUDONG?

A.12,097 feet.

B.12,093 feet.

C.13,123 feet.

A X

(Refer to Figure 6-3, 6-4) What distance is available for takeoff on Rwy 17 at

PUDONG?

A.12,093 feet.

B.12,097 feet.

C.13,123 feet.

C X

(Refer to Figure 6-4) For an aircraft of Category D with 4 turbo-powered

engines, what is the takeoff minimum for Rwy 17 at PUDONG with HIRL and CL

operating?

A.RVR 300m.

B.RVR 400m.

C.RVR 250m.

C X

(Refer to Figure 6-4) For an aircraft with 2 turbo-powered engines, what is the

takeoff minimum for Rwy 35 at PUDONG with only RL operating?

A.RVR 250m.

B.RVR 400m.

C.RVR 200m.

B X

(Refer to Figure 6-4) The color of Rwy 17 HIALS is

A.blue.

B.white.

C.green.

B X

(Refer to Figure 6-4) What is the takeoff minimum for Rwy 17 at PUDONG that

applies to an aircraft with 2 propelled engines?

A.VIS 1,600m.

B.RVR 200m.

C.RVR 250m.

A X

(Refer to Figure 6-5) At the bottom of this enroute chart excerpt, ZHENGZHOU

ACC's information is enclosed in a rectangle, what is the meaning of the

asterisk symbol preceding 122.20?

A.It means frequency 122.20 is very important.

B.It means VHF communication is available on a part-time basis.

C.It means 122.20 MHz is the major communication frequency.

Page 84: 1500 questions Very Important.pdf

B X

(Refer to Figure 6-5) If an aircraft is estimated to arrive WEIXIAN at 1300z, pilot

should contact ZHENGZHOU CONTROL on the frequency of

A.122.20MHz.

B.8897KHz.

C.3016KHz.

C X

(Refer to Figure 6-5) What is the announced FL on the airway B215 between

TAIYUAN and DAWANGZHUANG?

A.9,000m.

B.12,000m.

C.3,100m.

A X

(Refer to Figure 6-5) What is the MAA on the airway A461 between ZHOUKOU

and WEIXIAN?

A.FL 29,500 feet.

B.FL 8,100 feet.

C.FL 39,400 feet.

C X

(Refer to Figure 6-5) What is the MSA on the airway B215 between TAIYUAN

and DAWANGZHUANG?

A.FL 10,100 feet.

B.FL 8,100 feet.

C.FL 39,400 feet.

B X

(Refer to Figure 6-6) After takeoff from Rwy 36L at CAPITAL, if the flight is

cleared to depart via YV 4D, pilot must tune VHF NAV on the frequency of

A.114.7MHz.

B.514MHz.

C.113.6MHz.

C X

(Refer to Figure 6-6) Determine the VHF NAV frequency for the YV 14D after

takeoff from Rwy 36R at CAPITAL on

A.113.6KHz.

B.113.6MHz.

C.114.7MHz.

B X

(Refer to Figure 6-6) Using an average ground speed of 140 knots, what

minimum indicated rate of climb must be maintained to meet the required climb

gradient of 4.6% to 4,930 as specified on the SID?

A.690 feet/minute.

B.644 feet/minute.

C.600 feet/minute.

B X

Page 85: 1500 questions Very Important.pdf

(Refer to Figure 6-6) Using an average ground speed of 150 knots, what

minimum indicated rate of climb must be maintained to meet the required climb

gradient of 5% to 4,930 as specified on the SID?

A.760 feet/minute.

B.851 feet/minute.

C.699 feet/minute.

A X

(Refer to Figure 6-6) Using an average ground speed of 220 knots to depart

along VY2D, what minimum indicated rate of climb must be maintained to meet

the required climb gradient of 6.2% to FL148?

A.1,320 feet/minute.

B.1,400 feet/minute.

C.1,364 feet/minute.

C X

(Refer to Figure 6-6) What is the identifier of TANGHEKOU NDB?

A.YV.

B.WF.

C.DK.

A X

(Refer to Figure 6-7) For a CAT D aircraft arriving along 007o, the

pilot-in-command could descend it to

A.FL138.

B.FL128.

C.FL158.

C X

(Refer to Figure 6-7) How should the pilot identify the position to leave the area

control for the LR 2A?

A.GUBEIKOU NDB.

B.HUAIROU VOR/DME.

C.The descent moment on the course of 227o.

A X

(Refer to Figure 6-7) If an aircraft belongs to CAT B and is descending along the

course of 227o, which altitude could it descend to?

A.FL158.

B.FL148.

C.FL168.

B X

(Refer to Figure 6-7) If the en route altitude of an aircraft is FL118, what is the

altitude when the aircraft operates to DAWANGZHUANG along 007o

A.FL118.

B.FL128.

C.FL138.

A X

(Refer to Figure 6-7) The initial point of KM 2A is at

Page 86: 1500 questions Very Important.pdf

A.The point cleared by ATC.

B.D38 to HUAIROU.

C.HUAILAI.

C X

(Refer to Figure 6-7) The initial point of VYK 2A is at

A.D25 to PEK VOR/DME.

B.DAWANGZHUANG.

C.The point cleared by ATC.

B X

A pilot is flying in IFR weather conditions and has two-way radio

communications failure. What altitude should be used?

A. Last assigned altitude, altitude ATC has advised to expect, or the MEA,

whichever is highest.

B. An altitude that is at least 1,000 feet above the highest obstacle along the

route.

C. A VFR altitude that is above the MEA for each leg.

A X

A pilot is holding at an initial approach fix after having experienced two-way

radio communications failure. When should that pilot begin descent for the

instrument approach?

A. At the EFC time, if this is within plus or minus 3 minutes of the flight plan ETA

as amended by ATC.

B. At flight plan ETA as amended by ATC.

C. At the EFC time as amended by ATC.

C X

A pilot is operating in Class G airspace. If existing weather conditions are below

those for VFR flight, an IFR flight plan must be filed and an ATC clearance

received prior to

A.entering approach controlled airspace.

B.entering instrument meteorology conditions.

C.takeoff if weather conditions are below IFR minimums.

A X

A plane, MH 160°, receive this ATC clearance: "... HOLD EAST OF THE ABC

VORTAC ON THE ZERO NINER ZERO RADIAL...",What is the recommended

procedure to enter the holding pattern?

A.Teardrop only

B.Direct only

C.Parallel only

C X

A plane, MH 160°, receive this ATC clearance: "...CLEARED TO THE ABC

VORTAC HOLD SOUNTH ON THE ONE EIGHT ZERO RADIAL, LEFT

TURNS...",What is the recommended procedure to enter the holding pattern?

A.Teardrop only

B.Direct only

Page 87: 1500 questions Very Important.pdf

C.Parallel only

C X

A plane, MH 240°, receive this ATC clearance: "... HOLD WESTT OF THE ABC

VORTAC ON THE TWO SEVEN ZERO RADIAL...",What is the recommended

procedure to enter the holding pattern?

A.Teardrop only

B.Direct only

C.Parallel only

A X

A plane, MH 300°, receive this ATC clearance: "... HOLD WEST OF THE ABC

VORTAC ON THE TWO SEVEN ZERO RADIAL...",What is the recommended

procedure to enter the holding pattern?

A.Teardrop only

B.Direct only

C.Parallel only

C X

A plane, MH 60°, receive this ATC clearance: "... HOLD EAST OF THE ABC

VORTAC ON THE ZERO NINER ZERO RADIAL...",What is the recommended

procedure to enter the holding pattern?

A.Teardrop only

B.Direct only

C.Parallel only

A X

A provisional airport is an airport approved by the Administrator for use by an air

carrier certificate holder for the purpose of

A.obtaining provisions and fuel when unable, due to winds, to proceed direct to

the regular airport.

B.having the aircraft catered (foods, beverages, or supplies).

C.providing service to a community when the regular airport is unavailable.

C X

Activities, which are prohibited during critical phases of flight, include that

A. filling out logs

B. making passenger announcements

C. both a and b

C X

After experiencing two-way radio communications failure en route, when should

a pilot begin the descent for the instrument approach?

A. Upon arrival at any initial approach fix for the instrument approach procedure

but not before the flight plan ETA as amended by ATC.

B. Upon arrival at the holding fix depicted on the instrument approach

procedure at the corrected ETA, plus or minus 3minutes.

C. At the primary initial approach fix for the instrument approach procedure at

the ETA shown on the flight plan or the EFC time, whichever is later.

A X

Page 88: 1500 questions Very Important.pdf

An airport approved by the Administrator for use by an air carrier certificate

holder for the purpose of providing service to a community when the regular

airport is not available is a/an:

A.destination airport.

B.provisional airport.

C.alternate airport.

B X

An alternate airport for departure is required

A.if weather conditions are below authorized landing minimums at the

departure airport.

B.when the weather forecast at the estimated time of departure is for landing

minimums only.

C.when destination weather is marginal IFR.

A X

An alternate airport is not required for a supplemental or commercial air carrier,

turbojet-powered airplane on an IFR outside the contiguous China, if enough

fuel

A. is aboard to fly to the destination at normal cruise speed and thereafter at

least 2 hours at normal holding speed.

B. is aboard the airplane to fly to the destination and then to fly for at least 2

more hours at normal cruising fuel consumption.

C. to fly over the destination for 30 minutes at holding airspeed at 1,500 feet

AGL is carried aboard the airplane.

B X

An alternate airport must be listed in the dispatch and flight release for all

international operation air carrier flights longer than

A.7 hours.

B.8 hours.

C.6 hours.

C X

An ATC "instruction"

A.must be read back in full to the controller and confirmed before becoming

effective.

B.is a directive issued by ATC for the purpose of requiring a pilot to take specific

action.

C.is the same as an ATC clearance.

B X

Below that altitude, except when in cruise flight, are non-safety related cockpit

activities by flight crew members prohibited?

A. 3,000 meters.

B. 3,600 meters.

C. 6,000 meters.

A X

Class 2 NOTAMs could be divided into

Page 89: 1500 questions Very Important.pdf

A.Series A and Series C.

B.Series A, Series C and Series D.

C.Series A and Series D.

A X

During the period of Spring Festival, if you are assigned to an additional flight,

you should fill in the TYPE OF FLIGHT box with letter

A.N.

B.M.

C.X.

A X

Each pilot who deviates from an ATC clearance in response to a TCAS advisory

is expected to notify ATC and

A.Maintain the course and altitude resulting from the deviation, as ATS has

radar contact

B.Request a new ATC clearance

C.Expeditiously return to the ATC clearance in effect prior to the advisory , after

the conflict is resolved

C X

How many days ago does a Class 1 NOTAM must be disseminated before it

becomes effective?

A.7 days ago.

B.15 days ago.

C.28 days ago.

A X

How often is SNOWTAM broadcast to distant centers?

A.Hourly.

B.One hour and a half a time.

C.30 minutes a time.

A X

If a flight has not been cleared for approach while on a radar vector and it

becomes apparent that the current vector will take it across the final approach

course

A. the pilot should advise ATC of the situation. Do not turn to intercept the

approach course unless cleared to do so.

B. after advising ATC of the situation, the pilot may turn to intercept the

approach course.

C. the pilot may make a chance between A and B

A X

If a pilot is being radar vectored in IFR conditions and loses radio

communications with ATC, what action should be taken?

A. Fly directly to the next point shown on the IFR flight plan and continue the

flight.

B. Squawk 7700 and climb to VFR on Top.

C. Fly direct to a fix, route, or airway specified in the vector clearance.

Page 90: 1500 questions Very Important.pdf

C X

If a pilot suspects that he/she is suffering the effects of hypoxia, the most

appropriate remedy would be

A.use supplement oxygen or descend immediately to a low level

B.voluntarily increase the depth of breathing to induce more oxygen into the

lungs

C.voluntarily increase the breathing rate to increase the oxygen uptake

A X

If a received ATC clearance seems to be conflict to a regulation, the pilot should

A.Do not accept the clearance.

B.Read the clearance back entirely.

C.Request a clarification from the ATC immediately.

C X

If an aircraft is operating within Class B airspace at an indicated airspeed

greater than 250 knots, the elevation of the highest mountain from left 25

kilometers to right 25 kilometers along the route is 2,650 meters, the minimum

safe altitude is

A.3,250m.

B.3,050m.

C.2,950m.

A X

If an aircraft requests to land on a closed or unsafe runway, can the pilot

received landing clearance from ATC?

A. yes. At this time it is the pilot's responsibility for the landing safety if the pilot

decide to land.

B. no. At this time it is the pilot's responsibility for the landing safety if the pilot

decide to land.

C. no. the aircraft is prohibit for landing by the ATC.

B X

If an aircraft requests to land on a closed or unsafe runway, the pilot will be

informed by ATC as

A. no aircraft are allowed to land.

B. clear to land with caution.

C. runway is closed or unsafe runway.

C X

If an ATC controller assigns a speed, which is too fast for the aircraft's operating

limitation under the existing circumstance, what should the pilot do?

A. increase speed to the assigned speed.

B. increase speed to the maximum speed at that circumstance.

C. advise ATC of the speed that will be used and expect the controller issue

new speed.

C X

If the cruising speed of an aircraft is 0.70 Marh, pilot may enter the CRUISING

SPEED box with

Page 91: 1500 questions Very Important.pdf

A.M070.

B.070M.

C.M0070.

A X

If the cruising speed of an aircraft is 0.70 Marh, pilot may enter the CRUISING

SPEED box with

A.M070.

B.070M.

C.M0070.

A X

If the cruising speed of an aircraft is 420 nautical miles, pilot should fill in the

CRUISING SPEED block with

A.420N.

B.N420.

C.N0420.

C X

If the earliest takeoff time is 0800z, the first SNOWTAM must be sent out prior

to

A.0600z.

B.0630z.

C.0700z.

B X

If the estimated time of departure is 1400 Beijing Time, pilot should file flight

plan at least before

A.1330 Beijing Time.

B.1300 Beijing Time.

C.1230 Beijing Time.

C X

If the first portion of the flight is under IFR and the latter portion is under VFR,

pilot should fill the FLIGHT RULES box with letter

A.Y.

B.I.

C.Z.

C X

If the flight level of an aircraft is desired to be 10,200 meters, pilot should fill in

the LEVEL block of the flight plan with

A.F1020.

B.S1020.

C.L1020.

B X

If the flight level of an aircraft is desired to be 25,600 feet, pilot should fill in the

LEVEL block of the flight plan with

A.F256.

B.F0256.

Page 92: 1500 questions Very Important.pdf

C.M256.

A X

If the flight level of an aircraft is desired to be 9,000 meters, pilot should fill in

the LEVEL block of the flight plan with

A.M9000.

B.S9000.

C.M0900.

C X

If the landing minimums for a NDB approach shown on the IAP chart are

visibility 2,000m and MDA 120m, which minimums apply when you actually

divert to this airport?

A.Visibility 2,800m and MDA 180m.

B.Visibility 3,600m and MDA 240m.

C.Visibility 2,000m and MDA 120m.

C X

If the landing minimums for an ILS approach shown on the IAP chart are RVR

550m and DH 60m, which minimums apply when you actually divert to this

airport?

A.RVR 800m and DH 60m.

B.RVR 550m and DH 60m.

C.RVR 1,600m and DH 120m.

B X

If the pilot decided to land on a closed or unsafe runway in emergency, who is

responsible for the safety in controlled airport?

A. it is the pilot's responsibility for the operating of the aircraft and the

separations between other aircraft.

B. it is the pilot's responsibility for the operating of the aircraft and it is the ATC's

responsibility for the separations between other aircraft.

C. no one have the responsibility during emergency situation.

B X

If the wind direction measured by the weather station is 180°,the optimum

takeoff and landing direction is .

A.from south to north

B.from west to east

C.from north to south

C X

If the wind direction measured by the weather station is 270°,the optimum

takeoff and landing direction is .

A.from east to west

B.from west to east

C.from north to south

A X

7.生理学生理学生理学生理学,,,,人的因素人的因素人的因素人的因素,,,,CRM

Page 93: 1500 questions Very Important.pdf

Night vision is enhanced by

A.looking slightly to one side of the object you wish to view

B.fixing your gaze director on the object you wish to view.

C.ensuring that the cockpit remains brightly illuminated

A X

One of the effects of hypoxia is a degradation of night vision. This effect begins

at about

A.5000 feet

B.10000 feet

C.8000 feet

A X

Relying on the instruments and believing what they tell you, are the keys to

avoid vestibular illusion. The statement is

A.right

B.wrong

A X

Scanning procedures for effective collision avoidance should constitute

A.looking outside for 15 seconds, then inside for 5 seconds, then repeat.

B.1 minute inside scanning, then 1 minute outside scanning, then repeat.

C.looking outside every 30 seconds except in radar contact when outside

scanning is unnecessary.

A X

Some hazardous attitudes may affect pilot decision making. One of the

characteristic of "Anti-authority" is

A.refuses to listen to the advice or suggestions of others

B.adopt all rules, regulations, and procedures as far as possible

C.does the first thing that comes to mind.

A X

The air carrier must give instruction on such subjects as respiration, hypoxia,

and decompression to crewmember serving on pressurized airplanes operated

above

A. FL 180.

B. FL 200.

C. FL 250.

C X

The fuel reserve required, for a turbopropeller supplemental air carrier airplane

upon the arrival at a destination airport for which an alternate airport is not

specified, is

A. 3 hours at normal consumption no wind condition.

B. 3 hours at normal cruising fuel consumption.

C. 2 hours at normal cruising furl consumption.

B X

The fuel reserve required for a turbine-engine-powered (other than

turbopropeller) supplemental air carrier airplane upon arrival over the most

Page 94: 1500 questions Very Important.pdf

distant alternate airport outside the contiguous China is

A. 30 minutes at holding speed, at 1,500 feet over the airport.

B. 30 minutes, over the airport, at 1,500 feet, at cruising speed.

C. 2 hours at the normal cruising fuel consumption rate.

A X

The reserve fuel supply for a domestic air carrier flight is

A. 30 minutes plus 15 percent at normal fuel consumption in addition to the fuel

required to the alternate airport.

B. 45 minutes at normal fuel consumption in addition to the fuel required to fly to

and land at the most distant alternate airport.

C. 45 minutes at normal fuel consumption in addition to the fuel required to the

alternate airport.

B X

What is the fuel reserve requirement for a commercially operated

reciprocating-engine-powered airplane flying within contiguous China upon

arrival at the most distant alternate airport specified in the flight release?

Enough fuel to fly

A. 30 minutes plus 15 percent of total time required to fly at normal cruising

consumption to to the alternate.

B. to fly for 90 minutes at normal cruising fuel consumption.

C. 45 minutes at normal cruising fuel consumption.

C X

When a speed adjustment is necessary to maintain separation, what minimum

speed may ATC request of an aircraft operating above 3,000 meters, which

normal cruising speed is 350 knots?

A. 350 knots

B. 330 knots

C. 250 knots

C X

The average time of useful consciousness when engaged in moderate activity

following a rapid decompression at 30,000 feet is closest to

A.25 seconds

B.45 seconds

C.1 minute

B X

The blind spot is

A.the area of the lens which is screened by the iris.

B.at the junction of the optic nerve and the retina

C.an area on the cornea which does not respond to light.

B X

The brain perceives the body's orientation in space by

A.processing information from the eyes.

B.processing information from the inner ear.

C.by combining and comparing visual, vestibular and postural cues.

Page 95: 1500 questions Very Important.pdf

C X

The captain of a multi-crew aircraft rarely consults other members of the crew

and discourages any verbal input from them in the course of a flight. This

situation could be described as

A.a shallow authority gradient

B.a steep authority gradient

C.strong leadership

B X

The component of blood which is most directly involved in the transport of

oxygen throughout the body is

A.the red blood cells

B.the plasma

C.hemoglobin

C X

The Coriolis illusion can produce an unpleasant tumbling sensation. It is most

likely to occur when

A.the head is turned rapidly from left to right during visual flight

B.the head is bowed forward rapidly during instrument flight

C.an aircraft which has been in a shallow turn for some time rolls back to level

flight

B X

The decision-making process is quite complex; however, it can be condensed

into six elements, using the acronym DECIDE. The first D is

A.Detect the fact that a change has occurred

B.Do the necessary action to adapt to the change

C.Do what we must to do in the situation

A X

The decision-making process is quite complex; however, it can be condensed

into six elements, using the acronym DECIDE. The first E is

A.Estimate the other pilots' ability of dealing with this problem

B.Evaluate the effect of the action

C.Estimate the need to counter or react to the change

C X

The decision-making process is quite complex; however, it can be condensed

into the following six elements,

A.DICIDE

B.DECIDE

C.DECIED

B X

The function of the ciliary muscles in the eyes is to

A.move the eyes from left to right in the socket

B.change the size of the pupil to vary the amount of light entering the eye.

C.alter the shape of the lens to allow the eye to focus on objects at various

distances.

Page 96: 1500 questions Very Important.pdf

C X

The illusion of being in a noseup attitude which may occur during a rapid

acceleration take off is known as

A.inversion illusion.

B.autokinesis.

C.somatogravic illusion.

C X

The most effective way to deal with an episode of disorientation in flight is to

A.keep a constant look-out for glimpses of the natural horizon outside

B.rely absolutely on the aircraft instrument

C.keep the control column central and maintain a constant power

B X

The most effective way to scan the sky for other aircraft during level flight is to

A.move the head in a continuous arc from side to side.

B.do not look anywhere but straight ahead unless you detect movement.

C.move the head about 200 or 300 at a time, pausing after each movement to

allow the peripheral vision to detect any movement.

C X

The perception of color is a function of the

A.cones on the retina and is diminished in dim ambient lighting.

B.rods on the retina and is diminished in dim ambient lighting.

C.cones on the retina and is diminished in bright lighting conditions.

A X

The persons jointly responsible for the initiation, continuation, diversion, and

termination of a supplemental air carrier or commercial operator flight are the

A.pilot in command and chief pilot.

B.pilot in command and director of operations.

C.pilot in command and the flight follower.

B X

The pilot in command has emergency authority to exclude any and all persons

form admittance to the flight deck

A. except a FAA inspector doing enroute checks.

B. in the interest of safety.

C. except persons who have authorization from the certificate holder and the

CAAC.

B X

The position of leader and the follower may be changed in some special

conditions. While remaining overall leader of the flight crew, the captain might

not be the leader in a specific

A.(续正文,circumstance in which another member of the team has great

knowledge. The statement is)right

B.wrong

A X

The proprioceptive system generates sensation from

Page 97: 1500 questions Very Important.pdf

A.the semicircular canals

B.the skeletal muscles, joints and tendons

C.the otolith

B X

The somatogravic illusion (falsly identifying a level acceleration as a steep

climb), is most likely to be encountered

A.during a go-around in a high performance aircraft when flying visually

B.during and just after take-off in a high performance aircraft on a dark night

C.during an approach to a poorly lit runway on a dark night

B X

The static organ is responsible for the perception of linear acceleration. The

statement is

A.right

B.wrong.

A X

The time of useful consciousness (TUC) is the maximum time that pilot has to

make a rational, lifesaving decision and carry it out following

A.(续正文,a lack of oxygen at a given altitude. The statement is )right

B.wrong

A X

The time of useful consciousness while cruising at an altitude of 30,000 feet

and sitting quietly would be about

A.40 seconds

B.3 minutes

C.1 minute and 15 seconds

C X

The time of useful consciousness(TUC) is affected by many factors, such as

flight altitude, climbing rate, pilot's activity level, pilot's

A.(续正文,health, and whether the pilot smoking or not. The statement is )right

B.wrong

A X

The two different types of light sensitive elements on the retina are classified as

A.rods which are sensitive to color and cones which work best in dim light.

B.cones which are sensitive to color and rods which work best in dim light.

C.rods and cones both of which are responsible for color vision.

B X

What cause hypoxia?

A.Excessive carbon dioxide in the atmosphere.

B.An increase in nitrogen content of the air at high altitudes.

C.A decrease of oxygen partial pressure.

C X

What is the most effective way to use the eyes during night flight?

A.Look only at far away, dim lights.

B.Scan slowly to permit off-center viewing.

Page 98: 1500 questions Very Important.pdf

C.Concentrate directly on each object for a few seconds.

B X

What is the most effective way to use the eyes during night flight?

A.Scan slowly to permit off-center viewing.

B.Scan fast to permit off-center viewing.

C.Concentrate directly on each object for a few seconds.

A X

What illusion, if any, can rain on the windscreen create?

A.Does not cause illusions.

B.Lower than actual.

C.Higher than actual.

C X

What is a symptom of carbon monoxide poisoning?

A.Rapid, shallow breathing.

B.Pain and cramping of the hands and feet.

C.Dizziness.

C X

What is the effect of alcohol consumption on functions of the body?

A.Alcohol has an adverse effect, especially as altitude increases.

B.Small amounts of alcohol in the human system increase judgment and

decision-making abilities.

C.Alcohol has little effect if followed by equal quantities of black coffee.

A X

What type turbulence should be reported when it momentarily causes slight,

erratic changes in altitude and/or attitude, one-third to two-thirds of the time?

A. occasional light chop.

B. Moderate chop.

C. Intermittent light turbulence.

A X

When light passes through the lens it is brought to focus at the back of the

eyeball on the

A.retina

B.cornea

C.iris

A X

Which of the following is considered a primary flight control ?

A. Slats

B. Elevator

C. Dorsal fin

B X

Which of the following is considered an auxiliary flight control?

A. Ruddervator

B. Upper rudder

C. Leading-edge flaps

Page 99: 1500 questions Very Important.pdf

C X

Which of the following is not a symptom of carbon monoxide poisoning?

A.headache and fatigue

B.a feeling of euphoria

C.impairment of vision and mental confusion

B X

Which of the following is not normally a symptom of hypoxia?

A.increased visual field

B.An increase in breathing rate

C.Sleepiness or frequent yawning

A X

Which of the following is not normally a symptom of hypoxia?

A.several headache

B.reduced visual field

C.a feeling of euphoria

A X

Which of the following is not the clue of SA weaken or loss during the flight?

A.Fixation-focusing on any one thing to the exclusion of everything else.

B.Adhere to standard operating procedures.

C.Failure to meet expected checkpoint on flight plan or profile-ETA, fuel burn,

etC.as far as possible.

B X

Which of the following is not the right description about CRM?

A.CRM uses the crew as the unit of training.

B.CRM embraces some of operational personnel only.

C.CRM concentrates on crew members' attitudes and behaviors and their

impact on safety.

B X

Which of the following is not the tip for good SA management during the flight?

A.Resolve discrepancies- contradictory data or personal conflicts.

B.Fixation-focusing on any one thing to the exclusion of everything else.

C.Monitor and evaluate current status relative to our plan.

B X

Which of the following is the clue of SA weaken or loss during the flight?

A.Fixation-focusing on any one thing to the exclusion of everything else.

B.Adhere to standard operating procedures.

C.Meet expected checkpoint on flight plan or profile-ETA, fuel burn, etC.as far

as possible.

A X

Which of the following is the most likely cause of disorientation?

A.a change in the information coming from the inner ear.

B.a conflict or ambiguity in the information coming from visual, vestibular and

postural cues.

C.poor lighting reducing the amount of visual information received.

Page 100: 1500 questions Very Important.pdf

B X

Which of the following is the right description about CRM?

A.CRM is a comprehensive system of applying human factors concepts to

improve crew performance.

B.CRM is a comprehensive system of applying human factors concepts to

improve the captain's performance.

C.CRM is a comprehensive system of applying human factors concepts to

improve the pilots' performance.

A X

Which of the following is the right description about feedback in the

communication process?

A.Feedback is the check on how successful we have been in transferring our

message as ordinary intended.

B.We can use feedback to put the message back into the system as a check

against misunderstanding.

C.A and B

C X

Which of the following is the right description about the effect of alcohol

consumption on functions of the body ?

A.An individual can speed up the rate at which alcohol leaves the body.

B.Alcohol can affect the semi-circular canals, which leads to an increase in

susceptibility to disorientation and motion sickness.

C.Small amounts of alcohol in the human system increase judgment and

decision-making abilities.

B X

Which of the following is the tip for good SA management during the flight?

A.Focus on the details and scan the big picture.

B.Solicit input from all crew members including cabin, ATC, maintenance,

dispatch, etC.

C.A and B

C X

Which of the following organs mediates the sense of balance?

A.Eustachian tube

B.the cochlea

C.the semicircular canals

C X

Which of the following will increase susceptibility to hypoxia

A.smoking and alcohol

B.fatigue

C.both A and B

C X

Which of the following would be suitable treatment for a person suffering from

carbon monoxide poisoning?

A.administer oxygen

Page 101: 1500 questions Very Important.pdf

B.have them re-breathe their exhaled breath from a bag placed over the nose

and mouth

C.give plenty of water

A X

Which of the following would most likely lead to carbon monoxide contamination

of the cockpit atmosphere?

A.excessive use of carburetor heat

B.fly low in a very thick smoke haze

C.a leak in the engine exhaust system

C X

Which is a common symptom of hyperventilation?

A.Tingling of the hands, legs, and feet.

B.Increased vision keenness.

C.Decreased breathing rate.

A X

Which of the following techniques would contribute most to good term problem

solving performance?

A.stands steadfastly by your own aims and opinions and don't be swayed by

others

B.don't say nothing just to "keep the peace" when you are genuinely unhappy

about a planned course of action

C.trust you personal opinion, at the same time, adopt the other flight crews'

ideas partially

B X

Which procedural is recommended to prevent or overcome spatial

disorientation?

A.Reduce head and eye movement to the greatest possible extent.

B.Rely on the kinesthetic sense.

C.Rely entirely on the indications of the flight instruments.

C X

Which would most likely result in hyperventilation?

A.A stressful situation causing anxiety.

B.The excessive consumption of alcohol.

C.An extremely slow rate of breathing and insufficient oxygen.

A X

who is exposed to a high glare environment while on the ground (beach sand or

snow), which would help to enhance night vision?

A.don't read small print within one hour of flying.

B.wear good quality sunglasses during the day.

C.avoid using contact lenses

B X

"Authority gradient" is an important factor that can have an effect on

communication in a multi-crew environment. In a flight crew, an apt arranging is

A.the caption's power, ability, longevity and status are higher much more than

Page 102: 1500 questions Very Important.pdf

the other members of the flight crew.

B.the caption's power, ability, longevity and status are equal to the other

members of the flight crew.

C.the caption's power, ability, longevity and status is higher than the other

members of the flight crew in some short.

C X

"Do something quickly!", may be the expression of pilots who have the following

attitude?

A.Macho

B.Invulnerability:

C.Impulsivity:

C X

"The leans " is a state of disorientation which often occurs when

A.an abrupt change from a climb to straight and level flight

B.an aircraft which has been in a shallow turn for some time rolls back to level

flight

C.an abrupt recovery or a rapid correction is made.

C X

A common source of human error is the false hypothesis. Under certain

conditions this is more or less likely than at other times. From the following list,

select the situation least likely to

A.(续正文, result in a person arriving at a false hypothesis)when expectancy of

an event is high

B.after a period of intense concentration

C.during normal operation

C X

A good leader also has the ability to act as a good follower. The statement is

A.right

B.wrong

A X

A good team member will not agree with a plan of action just to "keep the

peace" if he/she is feeling uneasy about it. The statement is

A.right

B.wrong

A X

A pilot is more subject to spatial disorientation when

A.ignoring or overcoming the sensations of muscles and inner ear.

B.eyes are moved often in the process of cross-checking the flight instruments.

C.body sensations are used to interpret flight attitudes.

C X

A pilot who was worried about the condition of the left engine in a twin-engine

airplane experienced an engine failure on take -off and feathered the left engine,

only to find that it was the right

A.(续正文,engine which failed. This pilot has been a victim of)expectancy

Page 103: 1500 questions Very Important.pdf

producing a false hypothesis

B.expectancy producing a mental block

C.a motor program producing a false hypothesis

A X

After been in exposed to normal lighting, dark adaptation(night vision)returns

after a recovery period of about

A.30 minutes

B.1 hours

C.5 minutes.

A X

An absence of visible ground feature, such as when landing over water,

darkened areas or terrain made featureless by snow,

A.(续正文,can create the illusion that)the aircraft is at a lower altitude than is

the case, which may lead descends too low on approach.

B.the aircraft is at a higher altitude than is the case, , which may lead a pilot

descends too low on approach.

C.the aircraft is at a higher altitude than is the case, , which may lead a pilot

descends too high on approach.

B X

An authoritative leader

A.tends to make final decisions in an emergency, but seldom in other situations

B.tends to make all team decisions and controls all resources

C.may ultimately decide the team's actions, but seldom takes into consideration

the team members' experience, knowledge, and preferences.

B X

As a general rule, supplement oxygen should be provided and used by the pilot

in the day if the flight is above

A.15000 feet

B.10000 feet

C.8000 feet

B X

Carbon monoxide is dangerous because

A.it displace oxygen from the blood's red cells

B.it is highly acidic and attacks the lining of the lungs

C.it displaces oxygen from the lungs causing suffocation

A X

Crew resource management is a process using all available information and

resources, i.e. equipment, procedures and people, to achieve a safe and

A.(续正文,efficient flight operation. The statement is)right

B.wrong

A X

Deference is one of the "hazardous thoughts" which affect human behavior and

decision making. It refer to

A.the tendency to resist authority and object to regulation

Page 104: 1500 questions Very Important.pdf

B.the tendency to believe that "it can not happen to me"

C.the tendency to blame another person or circumstance for whatever goes

wrong

C X

During the flight, a good leader should not

A.communicate with the team and keep them informed of intentions before

acting on those decisions

B.can not recognize different behavioral styles and balance concern for people

with concern for performance.

C.involve the team in most decision making processes.

A X

During the flight, the position of leader and the follower may be changed in

some special conditions. The statement is

A.right

B.wrong

A X

Effective CRM has some characteristics, which of the following is right

description about it?

A.CRM is a process using all available information and resources, i.e.

equipment, procedures and people, to achieve a safe and efficient flight

operation.

B.CRM can be blended into all forms of aircrew training

C.A and B

C X

Empty Field Myopia is the tendency of the ciliary muscle to relax when there are

no close objects in the field of view.

A.(续正文,This causes the eye to take up a focal length of approximately)one to

two meters.

B.five meters.

C.infinity.

A X

Factors that can inhibit effective communication can be external or internal.

External factors include

A.high environmental noise levels

B.a busy work environment or mental discomfort

C.the receiver's expectancy

A X

Good leadership in a group is characterized by

A. high personal attention and high task orientation

B.medium personal attention and high task orientation

C.medium personal attention and medium task orientation

A X

Hazardous vortex turbulence that might be encountered behind large aircraft is

created only when that aircraft is

Page 105: 1500 questions Very Important.pdf

A. Developing lift.

B. Operating at high airspeeds.

C. Using high power settings.

A X

Heavy coffee or tea, can stimulate the central nervous system and produce an

increase in alertness and activity, produce anxiety and

A.(续正文,drastic mood swings at one time.. The statement is)right

B.wrong

A X

Hypoxia is always accompanied by

A. dizziness

B.a degradation in the performance of a pilot

C.vomiting

B X

Hypoxia is the result of which of these conditions?

A.Insufficient oxygen reaching the brain.

B.Excessive carbon dioxide in the bloodstream.

C.Limited oxygen reaching the heart muscles.

A X

Hypoxia may be caused by

A.fly with a head cold

B.breathing too quickly and/or too deeply for the requirements of the body

C.flying at an altitude where the partial pressure of oxygen is too low

C X

If a pilot whose eyes have fully adapted to darkness is exposed to a bright flash

of light, the time required for dark adaptation to be re-established is most likely

to be

A.3 minutes.

B.15 minutes.

C.30 minutes.

C X

If the authority gradient is too steep,

A.the first officer may be unlikely to contribute anything to the decision making

process—even when he is sure that the captain has made a mistake!

B.The first officer respects the captain's command status but feels free to

contribute.

C.the first officer may be likely to contribute anything to the decision making

process

A X

In a communication process, some characteristics of the sender may affect the

encode message. These conditions include

A.the sender's knowledge, personality, and the ability of he/she make use of the

language

B.the sender's skill, attitudes, knowledge, and the social culture system

Page 106: 1500 questions Very Important.pdf

C.the sender and the receiver's skill, attitudes, knowledge, and the workload

B X

In a decision-making process , a participatory leader

A.allows each team member to have a say and to participate in team

processes.

B.tends to make all team decisions and controls all resources

C.may ultimately decide the team's actions, but seldom takes into consideration

the team members' experience, knowledge, and preferences.

A X

In a highly successful effort to better analyze SA incidents, Mica Endsley

developed a 3-level taxonomy in 1995. Using the taxonomy, ASRS reviewed

113 SA

A.(续正文,incidents that were reported it and found 169 SA errors. Most Of

them were classified as)Level 1 — Failure to correctly perceive the situation

B.Level 2 — Failure to comprehend the situation

C.Level 3 — Failure to comprehend the situation into the future

A X

In flight, a good leader should

A.set high standards of performance by demonstrating a high level of personal

performance

B.involve the team in most decision making processes

C.A and B

C X

In order to make an effective communication, what type leader the captain

should be?

A.a participatory leader

B.an authoritative leader

C.a leader can balance act between the two types management style,

authoritative and participatory.

C X

In sensing the orientation of the body in space, the brain assigns the highest

priority to information coming from

A.the eyes

B.the inner

C.the proprioceptive system

A X

8.气象气象气象气象

1..In the SIGMET 8 of the Figure 3, which weather phenomenon that influence

fly be described.

A. Severe mountain wave, obscured thunderstorm with hail

B. Severe turbulence, embedded thunderstorms

C. Severe mountain wave, embedded thunderstorm with hail

C X

Page 107: 1500 questions Very Important.pdf

2..In which condition is possible to present very serious icing conditions for

protracted fligh?

A. associated with the smallest size of water droplet similar to that found in

low-level stratus clouds.

B. associated with thick extensive stratified clouds that produce continuous rain

such as altostratus and nimbostratus.

C. a cold-soaked aircraft descends into warm, moist air.

B X

3..Just prior to takeoff, the captain learns that an unstable fast cold front is

passing his destination airport, and he can realize that the current weather may

be

A.thunderstorm, shower and high wind

B.clear, high wind, good visibility

C.stratonimbus, light rain, poor visibility

A X

4..On the weather chart of ground, the area which has great difference in

temperature between cold and warm air mass is.

A.front

B.low pressure center

C.shear line

A X

5..Radiation fog usually appears in .

A. spring and summer.

B. winter and autumn.

C. summer and autumn.

B X

6..Sudden penetration of fog can create the illusion of

A.pitching up.

B.pitching down.

C.leveling off.

A X

7..The abbreviated plain language " WS WRNG " expressing

A. Wind shear warning

B. Aerodrome warning

C. Hazardous weather warning

A X

8..The adverse effects of ice, snow, or frost on aircraft performance and flight

characteristics include decreased lift and

A. increased thrust.

B. A decreased stall speed.

C. An increased stall speed.

C X

9..What is the approximate rate unsaturated air will cool flowing upslope?

A. 3℃ per 1,000 feet.

Page 108: 1500 questions Very Important.pdf

B. 2℃ per 1,000 feet.

C. 4℃ per 1,000 feet.

A X

10..The tropopause at middle latitude area usually reaches.

A. 8 to 9 km

B.10 to 12 km

C.15 to 17 km

B X

11..What feature is associated with a temperature inversion?

A. A stable layer of air.

B. An unstable layer of air.

C. Air mass thunderstorms.

A X

12..Which are the only cloud types forecast in the Terminal Aerodrome Forecast

A. Altocumulus

B. Cumulonimbus

C. Stratocumulus

B X

13..Which INITIAL cockpit indications should a pilot be aware of when a

constant tailwind shares to a calm wind?

A. Altitude increase; pitch and indicated airspeed decrease.

B. Altitude, pitch, and indicated airspeed decrease.

C. Altitude, pitch, and indicated airspeed increase.

C X

14..What are basic conditions to form a thunderstorm?

A. an unstable lapse rate and terrain lifting force

B. an unstable lapse rate, an initial lifting force, and sufficient water vapor.

C. sufficient water vapor and front area

B X

15..What feature is normally associated with the cumulus stage of a

thunderstorm?

A. beginning of rain at the surface

B. frequent lightning

C. continuous updraft

C X

16..To allow pilots of in-trail lighter aircraft to make flight path adjustments to

avoid wake turbulence, pilots of heavy and large jet aircraft should fly

A. Below the established glide path and slightly to either side of the on-course

centerline.

B. On the established glide path and on the approach course centerline or

runway centerline extended.

C. Above the established glide path and slightly downwind of the on-course

centerline.

B X

Page 109: 1500 questions Very Important.pdf

17..Which type of icing is associated with the smallest size of water droplet

similar to that found in low-level stratus clouds?

A. clear ice.

B. Frost ice.

C. Rime ice.

B X

18..The blowing dust or sand usually appear in

A. summer in northern areas.

B. spring in northern areas.

C. spring in southern areas.

B X

19..The Federal Aviation Administration's Flight Information Service Data Link

(FISDL) provides the following products:

A.METARS.SIGMETS, PIREP'S and AIRMETS.

B.SPECIS, SIGMETS, NOTAM'S, and AIRMETS.

C.Convective SIGMETS, PIREPS, AWW's, and adverse conditions.

A X

20..The flight crew learn that there is a stationary front over their destination

airport. Therefore, they can conceive the present weather at the destination as.

A.cumulus, shower, thunderstorm , good visibility

B.stratus, light rain, poor visibility cause by low clouds or fog

C.cirrus, radiation fogs or strong turbulence.

B X

21..The heat of the stratophere air is chiefly from .

A.the absorption of sun radiation by the water vapor

B.the absorption of sun ultraviolet ray radiation by the ozone

C.the absorption of ground radiation by the air.

B X

22..The heaviest icing that easy to accumulate in flight is associated with

A.in cumliform clouds

B.in high clouds, such as cirrus.

C.In altocumulus cloud.

A X

23..The clouds which can produce thunderstorm and hail are .

A.nimbostratus

B.cumulonimbus

C.altostratus.

B X

24..The property change of the airmass is chiefly due to.

A.the change of sun radiation

B.the property change of the underlying cushion

C.the movement of the weather system

B X

25..There is ribbon like precipitation under the clouds but it evaporated before

Page 110: 1500 questions Very Important.pdf

reaching the ground. This phenomenon is referred to as .

A.plume

B.virga

C. dowdraft

B X

26..Turbulence encountered above 15,000 feet AGL, not associated with cloud

formations, should be reported as

A. convective turbulence.

B. High altitude turbulence.

C. Clear air turbulence.

C X

27..What action is appropriate when encountering the first ripple of reported

clear air turbulence (CAT)?

A.Extend flaps to decrease wing loading.

B.Extend gear to provide more drag and increase stability.

C.Adjust airspeed to that recommended for rough air.

C X

28..What airport condition is reported by the tower when more than one wind

condition at different positions on the airport is reported?

A. Light and variable.

B. Wind shear.

C. Frontal passage.

B X

29..What characterizes a ground-based inversion?

A. Convection currents at the surface.

B. Cold temperatures.

C. Poor visibility.

C X

30..What condition is necessary for the formation of structural icing in flight?

A. flying in any clouds.

B. Flying in rain.

C. the temperature of the aircraft surface is 0oC or colder and supercooled

water drops.

C X

31..What condition produces the most frequent type of ground-or surface-based

temperature inversion?

A. The movement of colder air under warm air or the movement of warm air

over cold air.

B. Widespread sinking of air within a thick layer aloft resulting in heating by

compression.

C. Terrestrial radiation on a clear, relatively calm night.

C X

32..What is a characteristic of the troposphere?

A. It contains all the moisture of the atmosphere.

Page 111: 1500 questions Very Important.pdf

B. There is an overall decrease of temperature with an increase of altitude.

C. The average altitude of the top of the troposphere is about 6 miles.

B X

33..What is a feature of supercooled water?

A. the water drop sublimates to an ice particle upon impact.

B. The unstable water drop freezes upon striking an exposed object.

C. The temperature of the water drop remains at 0℃ until it impacts a part of the

airframe, then clear ice accumulates.

B X

34..What is a likely location of clear air turbulences?

A. in an upper trough on the polar side of a jetstream.

B. Near a ridge aloft on the equatorial side of a high pressure flow.

C. Downstream of the equatorial side of a jetstream.

A X

35..What is an important characteristic of wind shear?

A. it is primarily associated with the lateral vortices generated by

thunderstorms.

B. it usually exists only in the vicinity of thunderstorms, but may be found near a

strong temperature inversion.

C. it may be associated with either a wind shift or a wind speed gradient at any

level in the atmosphere.

C X

36..What is the expected duration of an individual microburst?

A. two minutes with maximum winds lasting approximately 1 minute.

B. one microburst may continue for as long as 2to 4 hours.

C. seldom longer than 15 minutes from the time the burst strikes the ground

until dissipation.

C X

37..What is the lowest cloud in the stationary group associated with a mountain

wave?

A. rotor cloud.

B. Standing lenticular.

C. Low stratus.

B X

38..What temperature condition is indicated if precipitation in the form of wet

snow occurs during flight?

A. the temperature is above freezing at flight altitude.

B. The temperature is above freezing at higher altitudes.

C. There is an inversion with colder air below.

A X

39..What wind condition prolongs the hazards of wake turbulence on a landing

runway for the longest period of time?

A. Direct tailwind.

B. Light quartering tailwind.

Page 112: 1500 questions Very Important.pdf

C. Light quartering headwind.

B X

40..When advection fog has developed, what may tend to dissipate or lift the

fog into low stratus clouds?

A. Temperature inversion.

B. Wind stronger than 15 konts.

C. Surface radiation.

B X

41..When flying over the nimbostratus clouds, the pilot finds that there are

uptowering clouds on the thick layer. Top of the towering clouds extends to

9000 meters high and are blown to one side by the strong wind like horse's

mane.

A.(接正文 Based on this situation, he predicts that there must be__in the sheet

clouds.) cumulonimbus clouds

B. altocumulus clouds

C. false cirrus clouds

A X

42..When takeoff or landing at the airport on the edge of thunderstorm region,

which of the following descriptive weather should be watchouted particularly?

A. visibility and low clouds

B. windshear at low altitude

C. hail and rainstorm

B X

43..When warm wet airflow flows to the cold ground, it will usually produce .

A. layer clouds

B. smooth airflow and advection fog

C. warm front clouds

B X

44..Where can the maximum hazard zone caused by wind shear associated

with a thunderstorm be found?

A. in front of the thunderstorm cell (anvil side) and on the southwest side of the

cell.

B.. ahead of the roll cloud or gust front and directly under the anvil cloud.

C. on all sides and directly under the thunderstorm cell.

C X

45..Where do squall lines most often develop?

A.in an occluded front

B.ahead of a cold front

C.behind a stationary front

B X

46..Which condition is present when a local of air is stable?

A. The parcel of air resists convection.

B. The parcel of cannot be forced uphill.

C. As the parcel of air moves upward, its temperature becomes warmer than

Page 113: 1500 questions Very Important.pdf

the surrounding air.

A X

47..Which condition will for the formation of advection fog?

A. Moist, stable air being moved over gradually rising ground by a wind

B. A clear sky, little or no wind, and high relative humidity.

C. Moist air moves over colder ground or water.

C X

48..Which is a necessary condition for the occurrence of a low-level

temperature inversion wind shear?

A.the temperature differential between the cold and warm layers must be at

least 10℃.

B.a calm or light wind near the surface and a relatively strong wind just above

the inversion.

C.a wind direction difference of at least 30℃ between the wind near the surface

and the wind just above the inversion.

B X

49..Which of the following area has the highest probability to produce low level

windshear?

A.Area with strong cold advection

B.Area where the sea and the land meet

C. Area near the front with thunderstorms

C X

50..Which of the following cases illustrates airplane's entry one type of air mass

to another one?

A. The airplane flies through high level turbulence

B. Obvious change of temperature and wind direction within a short period of

time

C. The airplane meets rolling airflow when crossing mountainous areas

B X

51..Which of the following cases provides impulse force for forming heat

thunderstorm?

A. Water vapor coagulating and then releasing latent heat

B. Rising terrain

C. Uneven heat on the ground

C X

52..Which of the following characteristics heat thunderstorm?

A.It moves with the weather system.

B.It covers small area and is isolated with openings

C. It strengthens at night and weakens in the day on land

B X

53..Which of the following differs most greatly at the two sides of the front?

A.wind speed

B.air temperature

C.air pressure

Page 114: 1500 questions Very Important.pdf

B X

54..Which INITIAL cockpit indications should a pilot be aware of when a

headwind shears to a calm wind?

A. indicated airspeed decreases, aircraft pitches up, and altitude decreases

B. indicated airspeed increases, aircraft pitches down, and altitude increases

C. indicated airspeed decreases, aircraft pitches down, and altitude decreases

C X

55..which of the following statements about "hailstone" is correct?

A.ground rain means that the hailstone disappears in the air

B.towering clouds usually produce hailstones

C.the hail can be thrown several kilometers out of the thunderstorm.

C X

56..Which of the following weather phenomenon symbols that thunderstorm is

at its mature phase?

A. The appearance of disordered low clouds

B. The appearance of anvil clouds top

C. It begins to rain on the ground

C X

57..Which process causes adiabatic cooling?

A. Expansion of air as rises.

B. Movement of air over a colder surface.

C. Release of latent heat during the vaporization process.

A X

58..Which term applies when the temperature of the air changes by

compression or expansion with no heat added or removed?

A. Katabatic.

B. Advection.

C. Adiabatic.

C X

59..Which type clouds are indicative of very strong turbulence?

A. nimbostratus.

B. Standing lenticular.

C. Cirrocumulus.

B X

60..Which type of weather conditions are covered in the Convective SIGMET

A. Embedded thunderstorms, severe turbulence

B. Cumulonimbus clouds, light turbulence

C. severe icing, surface visibility lower than 5000 meters

A X

61..Which type precipitation is an indication that supercooled water is present?

A. wet snow.

B. Freezing rain.

C. Ice pellets.

B X

Page 115: 1500 questions Very Important.pdf

62..Which types of weather conditions are covered in the Convective SIGMET

A. Strong wind, volcanic ash

B. Fog, embedded thunderstorms with hail

C. Severe mountain wave, isolated cumulonimbus

C X

63..Which wind-shear condition results in a loss of airspeed?

A. Headwind or tailwind decrease

B. Decrease headwind and increasing tailwind

C. Increasing headwind and decreasing tailwind.

B X

64..Which wind-shear condition results in an increase in airspeed?

A. Increase tailwind wind and decreasing headwind.

B. Headwind or tailwind increase

C. Increasing headwind and decreasing tailwind.

C X

65..(According to Figure 1) How much is the vertical visibility at Chengdu

(ZUUU)

A. 50 metres

B. 500 metres

C. 150 metres

C X

66..(According to Figure 1) How much is the visibility at Shanghai (ZSSS)

A. Potential visibility is 3200 meters

B. Visibility is 10 kilometers or the above

C. Visibility is 5 kilometers or the above;

B X

67..(According to Figure 1) How much is the visibility at Urumchi (ZWWW)

A. 300 metres

B. 900 metres

C. 3000 metres

A X

68..(According to Figure 1) which station reports the max wind speed

A.ZSSS

B.ZUUU

C.ZWWW

C X

69..(According to figure 2) , which of the following forecast stations predicts the

icing?

A. ZSSS

B. ZUUU

C. ZHHH

B X

70..(According to figure 2) , which of the following forecast stations predicts the

turbulence?

Page 116: 1500 questions Very Important.pdf

A. ZUUU

B. ZHHH

C. ZSSS

C X

71..(according to figure 2) The weather at ZSSS before 0500Z is ___.

A. Scattered clouds, visibility 5000m and fog

B. West wind at 4 m/s, mist and broken clouds

C. East windat 4 m/s, rain and broken clouds

B X

72..(according to figure 2), at ZBAA, the max predicting wind speed is___.

A. 17 m/s

B. 8 m/s

C. 6 m/s

A X

73..(according to figure 2), the forcasting visibility at ZSSS is____

A. 5000 m between 00Z-05Z, and 3000 m at other time

B. 5000 m between 00Z-09Z

C. 5000 m between 00Z-05Z, 5000 m above at other time

A X

74..(according to figure 2)An airplane is estimated to arrive at ZUUU at 1000Z,

and the weather then is ____.

A. Light fog, visibility 4 km.

B. Southwest wind at 6 m/sec, light rain shower, visibility 4 km

C. Southeast wind at 24 m/sec, heavy rain, visibility 4000 m

B X

75..(According to figure 4) at Lanzhou, which description is correct

A. Smoke with overcast

B. Strong wind with sand storm

C. Strong wind and total sky obscuration

C X

76..(According to figure 4) at Lanzhou, which weather phenomenon that

influence fly be described

A. Strong wind, bad visibility

B. Dust, thunderstorm,

C. Low cloud with strong wind

A X

77..(According to figure 4) at Shanghai, which description is correct

A. The visibility is 800 meters, the sea level pressure is 996 hPa,

B. The visibility is 8 kilometers, the QNH is 999.6 hPa,

C. The visibility is 8 kilometers, the sea level pressure is 999.6 hPa,

C X

78..(According to figure 4) at Shanghai, which weather phenomenon that

influence fly be described.

A. Low cloud and perhaps icing

Page 117: 1500 questions Very Important.pdf

B. Strong wind with bad visibility

C. Snow shower

A X

79..(According to figure 4) The cloud condition at Shanghai is

A. There are altocumulus , stratocumulus, the bass of the stratocumulus is

600m

B. There are altostratus, stratocumulus, the bass of the stratocumulus is 1

200m

C. There are cirrostratus , stratus , the bass of the stratus is 1 200m

B X

(According to figure 4) The weather phenomenon at Lanzhou is

A. Smoke

B. Sand

C. Sand storm

B X

(According to figure 4) The weather phenomenon at Shanghai is

A. Rain and snow

B. Drizzle

C. Continuous snow

A X

(According to figure 6) At point B, the wind direction, wind speed and

temperatures is

A. Southwest wind at 14 KT, temperatures is -52℃

B. Northeast wind at 35 KT, temperatures is 52℃

C. Southwest wind at 35 KT, temperatures is -52℃

C X

(According to figure 6) The position of the upper jet is

A. From A to B then to C

B. From A to C then to D

C. From A to B then to D

B X

(According to figure 6) What changes take place from point B fly to point D

A. The wind speed increases, temperature rises

B. The wind speed increases, temperature reduced

C. The wind speed and wind direction is no changes, temperature reduced

B X

(Refer to Figure 1) According to the aeronautical weather report on figure 1, the

QNH for ZUUU is

A.998 hPa

B.1099.8 hPa

C.999.8 hPa

A X

(Refer to Figure 1) What weather condition is reported at Guangzhou (ZGGG)

A. Heavy thunderstorm with shower of rain and towering cumulus clouds

Page 118: 1500 questions Very Important.pdf

B. Heavy thunderstorm with rain and cumulonimbus clouds

C. Bad visibility and strong wind

B X

(Refer to Figure 1) What weather condition is reported at Urumchi (ZWWW)

A. Strong wind with shower

B. Snow and rain

C. Strong west wind and blowing snow

C X

(Refer to Figure 1) which station has the best visibility

A.ZBAA

B.ZSSS

C.ZUUU

B X

(Refer to Figure 3) In the telegram (3), which weather phenomenon that

influence fly be described.

A. The surface wind is heavier than the wind at heights of 60 m above runway

level

B. There are wind shears from place of 60 meters of end of the runway

C. There are wind speed and wind direction shears between the surface and

the height of 60 meters

C X

(Refer to Figure 5) Flying from Shenzhen to Zhengzhou, which weather

phenomenon that may be meet

A. Rain shower and thunderstorm

B. Rain and fog

C. Strong wind and thunderstorm

A X

(Refer to Figure 5) Flying from Shenzhen to Zhengzhou, which weather system

we coursed

A. It is a warm front.

B. It is a stationary front

C. It is a Cold frongt

C X

(Refer to Figure 5) What meaning is expressed on the area arrow K pointed

A. The area arrow K pointed is a center of low pressure

B. For past three hours, the air pressure have reduced 1.7hPa.

C. For past three hours, the air pressure have reduced 17hPa.

B X

(Refer to Figure 5) what weather area is there in the east of zhengzhou

A. Rain

B. Light fog

C. Fog

C X

(Refer to Figure 5) what weather phenomenon is there in the north of

Page 119: 1500 questions Very Important.pdf

zhengzhou

A. North wind at 12 m/s, dust

B. Strong wind with sand

C. Strong wind with haze

B X

(Refer to Figure 6) At point A, the wind direction, wind speed and temperatures

is

A. West wind at 60 KT, temperatures is -48℃

B. Northwest wind at 60 KT, temperatures is 48℃

C. West wind at 24 KT, temperatures is -48℃

A X

(Refer to Figure 7) Flying from Guangzhou to Chengdu, which significant

weather phenomenon may be encounterd

A. Thunderstorm, Rain shower, mountain waves, moderate clear air turbulence

B. Rain shower, freezing precipitation, mountain waves, severe clear air

turbulence

C. Rainfall, freezing precipitation, mountain waves, severe icing

B X

(Refer to Figure 7) Flying from Guangzhou to Chengdu, which weather system

we would encounter

A. A cold front and a upper jet

B. A worm front and a upper jet

C. Mountain waves and severe turbulence

A X

(Refer to Figure 7) Flying from Kunming to Chengdu, which significant weather

phenomenon may be encounterd

A. Severe turbulence, moderate icing, rain shower, thunderstorms

B. Moderate turbulence, severe icing, freezing precipitation, severe clear air

turbulence

C. Moderate turbulence, moderate icing, rain, severe clear air turbulence

C X

(Refer to Figure 7) Flying from Kunming to Chengdu, which weather system we

would encounter

A. A worm front and mountain waves

B. A cold front and moderate icing

C. A stationary front and a upper jet

C X

(Refer to Figure 7) In this chart, ISOL EMBD CB 340/XXX means

A. Some places have CBs, CBs with little or no separation

B. Isolated cumulonimbus embedded in layers of other clouds, the top of the

cumulonimbus is 34000 feet, the base is unknown

C. The airplane is embedded in layers of clouds, the top of the cumulonimbus is

34000 feet, the base is unknown

B X

Page 120: 1500 questions Very Important.pdf

(Refer to Figure 7) In this chart, ISOL EMBD CB 340/XXX means

A. Some places have CBs, CBs with little or no separation

B. Isolated cumulonimbus embedded in layers of other clouds, the top of the

cumulonimbus is 34000 feet, the base is unknown

C. The airplane is embedded in layers of clouds, the top of the cumulonimbus is

34000 feet, the base is unknown

B X

(Refer to Figure 7) The height and speed of the upper jet is

A. The height of the upper jet is 3500 feet, the wind speed is 120 km/h

B. The height of the upper jet is 35000 feet, the wind speed is 120 knots

C. The height of the upper jet is 35000 feet, the wind speed is 48 km/h

B X

____prevents air flowing directly from the high pressure area to low pressure

area when air moves in a level direction.

A. Friction force.

B. Inertial centrifugal force

C. Coriolis deflecting force

C X

A clear area in a line of thunderstorm echoes on a radar scope indicates

A. the absence of clouds in the area

B. an area of no convective turbulence

C. an area where precipitation drops are not detected

C X

Airplanes may encounter heavy turbulence in

A. Cumulonimbus and towering cumulus

B. Stratocumulus and altocumulus

C. Altocumulus and cumulonimbus

A X

An aircraft that encounters a headwind of 40 knots, within a microburst, may

expect a total shear across the microburst of

A. 40 knots

B. 80 knots

C. 90 knots

B X

An aircraft that encounters a headwind of 45knots, within a microburst, may

expect a total shear across the microburst of

A. 40 knots

B. 80 knots

C. 90 knots

C X

At lower levels of the atmosphere, friction causes the wind to flow across

isobars into a low because the friction.

A. decreases windspeed and Coriolis force.

B. decreases pressure gradient force.

Page 121: 1500 questions Very Important.pdf

C. creates air turbulence and raises atmospheric pressure.

A X

At which location does Coriolis force have the least effect on wind direction?

A. At the poles.

B. Middle latitudes(30o to 60o).

C. At the equator.

C X

before takeoff, the pilots find the clouds have the following: very low and thick

clouds, flat and gray bottoms with light rain. Thisclouds is referred as

A. stratus or nimbostratus

B. altostratus or stratocumulus

C. cumulonimbus or altocumulus

A X

Before taking off, the captain finds that on weather map, a stable cold front is

slowly moving towards his arrival airport,

A.(接正文 he should predict that the weather at the arrival airport might be)

cloudy with drizzle, low clouds and weak turbulent current

B. cumulonimbus clouds, shower and gale

C. clear after rain, north deflecting wind and strong turbulent current

A X

Convective clouds which penetrate a stratus layer can produce which threat to

instrument flight?

A. freezing rain

B. clear air turbulence

C. embedded thunderstorms

C X

During the life cycle of a thunderstorm, which stage is characterized

predominately by downdrafts?

A. cumulus

B. dissipating

C. mature

B X

How are smoke cleared or dispersed?

A. By wind or the movement of air.

B. By convective mixing in cool night air.

C. By evaporation similar to the clearing of fog.

A X

How can the stability of the atmosphere be determined?

A. Ambient temperature lapse rate.

B. Atmospheric pressure at various levels.

C. Surface temperature/dewpoint spread.

A X

How can turbulent air cause an increase in stalling speed of an airfoil?

A.An abrupt change in relative wind.

Page 122: 1500 questions Very Important.pdf

B.A decrease in angle of attack.

C.Sudden decrease in load factor.

A X

How does Coriolis force affect wind direction in the Southern Hemisphere.

A. Causes clockwise rotation around a low.

B. Causes wind to flow out of a low toward a high.

C. Has exactly the same effect as in the Northern Hemisphere.

A X

How dose Coriolis affect wind direction in the northern Hemisphere?

A. Causes counterclockwise rotation around a low.

B. Causes wind to flow out of a low toward a high.

C. Has exactly the same effect as in the Southern Hemisphere.

A X

If airplane enters to the undershoot airflow under thunderstorm clouds, it will

encounter___.

A. head windshear

B. turbulence

C. vertical windshear

C X

If the aircraft enters the strong downdraft burst, where is the aircraft expected to

meet the strongest downdraft?

A. in the center of strong downdraft burst

B. when it enters the strong downdraft burst

C. when leaves the strong downdraft burst

A X

If the airplane flies through a high pressure center in the northern hemisphere,

the wind direction usually changes.

A. from headwind to tailwind.

B. from left crosswind to right crosswind.

C. from right crosswind to left crosswind

C X

If the airplane fly through a low pressure center in the northern hemisphere, the

wind direction usually changes .

A.from left crosswind to right crosswind.

B.from headwind to tailwind.

C.from right crosswind to left crosswind

A X

If the ambient temperature is colder than standard at FL310, what is the

relationship between true altitude and pressure altitude?

A.They are both the same 31,000feet.

B.True altitude is lower than 31,000feet.

C.Pressure altitude is lower than true altitude.

B X

If the ambient temperature is warmer than standard at FL350, what is the

Page 123: 1500 questions Very Important.pdf

density altitude compared to pressure altitude?

A.Lower than pressure altitude.

B.Higher than pressure altitude.

C.Impossible to determine without information on possible inversion layers at

lower altitudes.

B X

In average, for every 1,000 feet increase in the troposphere, the temperature of

the air will .

A. descend 2℃

B.descend 3℃

C. descend 4℃

A X

In daily aeronautical weather reports, the code "VCTS" means ____.

A. There are thunderstorms in the vicinity of the airport but no thunderstorms

over the airport

B. There are rains in the vicinity of the airport but no thunderstorms over the

airport.

C. There are thunderstorms over the airport.

A X

In en route flight, the altimeter setting is

A. QNE

B. QNH

C. QFE

A X

In flight, pilots find forane clouds consisting of cloud bar and cloud band with flat

floor and piled clouds on the top of some clouds. The descriptive cloud should

be.

A. cumulus congestus

B. cumulostratus

C. towering clouds

C X

In northern hemisphere, if fly from high pressure area to low pressure area at

6000 meters, the wind on the course is _____.

A.right crosswind

B.tailwind

C. left crosswind

C X

In northern sphere, the air cyclone circumfluence is

A. outward, downward and clockwise

B. outward, upward and clockwise

C. inward, upward and counterclockwise

C X

In stable air associated with a high pressure system usually occurs?

A. Advection fog.

Page 124: 1500 questions Very Important.pdf

B. Radiation fog.

C. Frontal fog.

B X

In summer, when warm air mass is unstable, what may occur in layer clouds of

cold front?

A. cumulonimbus

B. front fog

C. layer clouds

A X

In the AIRMET 6 of the Figure 3, which weather phenomenon that influence fly

be described.

A. Lines of thunderstorms, fog and rain

B. Nimbostratus, obscured thunderstorms

C. Low ceiling cloud and moderate icing

C X

In the dark, a stationary light will appear to move when stared at for a period of

time. This illusion is known as

A.somatogravic illusion.

B.ground lighting illusion.

C.autokinesis.

C X

In the friction layer of the northern hemisphere, the wind of low pressure area

is .

A. clockwise inbound wind

B.counter clockwise outbound wind

C.counter clockwise inbound wind

C X

In the Northern hemisphere, if a pilot makes a long-distance flight from east to

west, under which of the following conditions,

A.(接正文 might encounter tailwind forming by air pressure system?) when the

airplane is in the south of high pressure system and in the north of low pressure

system.

B. When the airplane is in the north of high pressure system and in the south of

low pressure system.

C. When the airplane is in the north of high pressure and low pressure

systems .

A X

In the SIGMET 8 of the Figure 3, what is the valid period of time.

A. From 08Z to 22Z.

B. From the 22nd 08Z to the 22nd at 14Z.

C. From 14Z to 22Z

B X

In the SIGMET 8 of the Figure 3, which station predict the dangerous weather at

first

Page 125: 1500 questions Very Important.pdf

A. ZULS

B. ASSS

C. ZUUU

A X